Вы находитесь на странице: 1из 174

Page |1

Bergen Community College


Physical Sciences Department

Introduction to Chemistry
Course Number: CHM100

LABORATORY MANUAL 2017

Third Edition

Contributors

Dr. Frank Ramdayal Dr. Ara N. Kahyaoglu


Associate Professor Associate Professor

Bergen Community College


400 Paramus Road, Paramus, NJ 07652.
Page |2

PREFACE

To the instructor,

This manual is available as a free download from the Bergen Community College website. The
Science Department’s aim has been to provide low cost, safe, and interesting experiments.

Students are to complete the pre-laboratory examples prior coming to laboratory.

Much effort has been made by the chemistry faculty to review this manual and make it as error-free
and accurate as possible. However, some errors will have escaped our notice. Your help in
forwarding to us any errors, inaccuracies, and/or suggestions will be greatly appreciated. We will
definitely welcome your comments and suggestions. We will make the changes and improvements as
soon as we can to make the updated manual ready for succeeding semesters. We can be contacted
at akahyaoglu@bergen.edu and framdayal@bergen.edu.

Good luck, have a good semester. We look forward to hearing from you.

To the students,

This manual is available to BCC students for free download from the Bergen Community College
website. The Science Department’s aim has been to provide low cost, safe, and interesting
experiments that illustrate the concepts presented in your textbooks and lectures.

Safety is everyone’s number one priority. Do not hesitate to ask your instructor if you do not
understand the procedure.

Keep in mind that your instructor expects you to be prepared for every laboratory session.

We appreciate the efforts of Dr. P. J. Ricatto (Dean, Science and Mathematics), Dr. Linda Box
(Chairwoman, Science), Dr. Brent Chapman, Dr. Karen Comai-Fuerherm, Dr. Gary Porter, Dr. F.
Ramdayal and Dr. J. Tscherne for their suggestions and corrections. This work is financially
supported by a STEM grant.

Special thanks to Dr. Bruce Benjamin and Dr. Michael Ansonoff for reviewing the final manuscript.

Dr. Ramdayal and Dr. Kahyaoglu


Page |3

TABLE OF CONTENTS
Page
Check–In, Safety and Math Review 5

Experiment 1. Metric System and Measurements. 10

Experiment 2. Physical and Chemical Changes. 27

Experiment 3. Density of objects. 39

Experiment 4. Specific Heat of a Metal 51

Experiment 5. Flame Tests and Chemical Fingerprinting. 63

Experiment 6. Lewis Dot Structures and VSEPR Models 74

Experiment 7. Qualitative Analysis of Anions. 87

Experiment 8. Water of Hydration. 100

Experiment 9. Types of Chemical Reactions. 113

Experiment 10. Empirical Formula. 128

Experiment 11. Percent Yield of a Reaction. 140

Experiment 12. Paper Chromatography. 150

Experiment 13. Oxidation-Reduction. 160

Experiment 14. Solutions and pH. 175

Periodic Table 185

Prefixes and Conversion Factors 186


Page |4

Figure. Common Laboratory equipment.


Page |5

BERGEN COMMUNITY COLLEGE


SAFETY REGULATIONS FOR THE CHEMISTRY LABORATORY

1. Read these safety regulations carefully and be sure you understand them. Before each laboratory session,
your instructor will discuss any safety hazards that might be associated with that day’s experiment.
Therefore, it is imperative that you come to lab on time.

2. Due to safety concerns students who arrive after the pre-lab presentation may not be allowed to perform
that particular lab experiment.

3. It is strongly suggested that you obtain a hall locker from the Security Office. Only your lab manual,
notebook, and calculator are allowed on the lab bench.

4. Report all accidents, no matter how minor, to your instructor at once. No one in the laboratory is
permitted to give out bandages or medication. You must see the College Nurse.

5. Safety glasses or goggles are required and must be worn by everyone in the lab when experiments are
being conducted. Safety glasses are provided by the college, but students may purchase their own.

6. Do not perform any unauthorized experiment.

7. Do not taste anything in the laboratory. Never eat, drink or smoke in any of the laboratories.

8. You must tie back long hair. Do not wear open-toed shoes, shorts, fuzzy sweaters, loose sleeve shirt or
any dangling jewelry. You must cover bare midriffs. You are advised to wear a lab coat or old clothing
to the lab.

9. Do not fill pipettes by mouth. Rubber bulbs or pipette pumps are provided. The instructor will
demonstrate how these are to be used.

10. Exercise care when noting the odor of fumes. Use ‘wafting’ if you are directed to note an odor.

11. Do not force glass tubing or a thermometer into rubber stoppers. Lubricate the tubing with water and
introduce it gradually and gently into the stopper or insert through a cork borer. Protect your hands with
toweling when inserting without a cork borer.

12. Never point a test tube containing a reaction mixture (especially when heating) toward yourself or
another person.

13. No ‘fooling around’ in the laboratory. A less than serious approach to lab work may result in an
accident.

14. Before connecting or disconnecting electrical equipment, make sure that the switches are in the off
position,

15. Never work in the laboratory alone.


Page |6
16. Make sure all apparatus is properly supported on the workbench.

17. Read the label on every bottle twice before using it in the laboratory. Many chemical names are very
similar but are very different chemically.

18. Replace caps and stoppers on bottles immediately. Return spatulas to their correct place immediately
after use. Do not mix them up.

19. Do not remove or relocate any chemical that has been placed in the hood. Sample it in the hood.

20. Never light a Bunsen burner with a cigarette lighter. Use the strikers that are provided.

21. Students are responsible for keeping their work area neat and orderly. All spills are to be cleaned up
immediately using the spill kits located on the instructor’s desk. Solid chemical waste should be and
placed in the appropriately labeled container. Liquid chemical waste should be poured into the
appropriately labeled container. All waste material should be left in the hood for subsequent disposal. If
there is doubt about proper disposal, ask the instructor.

22. Wash all glassware immediately after use. Place clean glassware on drying rack or in the designated bin
on the counter.

23. Dispose of broken glassware in the labeled broken glassware boxes.

24. Wash your hands before leaving the laboratory.

25. You must notify your instructor of any chemical to which you are allergic.

26. If you are pregnant or planning to become pregnant this semester, you must notify your physician that
you are enrolled in a chemistry lab course. You and your physician must decide whether or not it is
appropriate for you to remain in the course.

Note the location of the following safety equipment so that you can get to it quickly in an emergency…

SAFETY EQUIPMENT LOCATION


FIRE EXTINGUISHER
SAFETY SHOWER
EYEWASH
EMERGENCY PHONE
FIRE ALARM
NEAREST EXIT
Page |7
Please sign this attached form. Remove it from the safety regulations and hand it to your Laboratory Instructor.

I, the undersigned, have read the Divisional Safety Regulations for the Chemistry Laboratories. I understand
them and will abide by them.

Print your name: ___________________________________________________________

Signature: _________________________________________________________________

Date: ____________________________________________________________________

Course Name and Number: ____________________________________________________

SAFETY IN THE LABORATORY True False

1. Safety glasses must be worn by everyone working in the lab. T F

2. Only major accidents in the lab need to be reported T F

3. Material Safety Data Sheets are provided in the lab T F

4. Eating and drinking are permitted in the lab T F

5. It is OK to taste a chemical as long as it smells good T F

6. Only authorized experiments are to be performed T F

7. You should wear shoes at all time in the lab T F

8. In order to save time, it is permissible to weigh hot objects T F

9. Broken glassware should be disposed of in the appropriate box T F

10. Working alone in the lab is an acceptable practice T F

A typical Chemistry Laboratory safety YouTube video link is given below: (hold Ctrl Key and hover the
mouse over the link) https://www.youtube.com/watch?v=UKovNdse5MU
Page |8

Math Assignment
Name______________________ CHM100 Section ___________

1. Solve the following:


a) 256 × 0.05 =

b) 2.5 × 100 =

c)

d)

e)

2. Express the following exponentially:


a) 10,000 c) 0.0015

b) 150 d) 0.023

3. Express the following as whole numbers:

a) c)

b) d)

4. If

a) Write an equation for c in terms of a and b

b) Write an equation for b in terms of a and c

5. Given that

a) Find

b) Find
Page |9
6. Given the following data, fine the average

35.3 35.6 34.9 35.0

7. 2.5 centimeters = 1.00 inches

a) How many centimeters are there in 5.0 inches?

b) How many inches are there in 21.0 centimeters?

8. The density is defined as the mass of a substance divided by its volume. Determine the density of
9.0 mL of a liquid with a mass of 13.4 grams.

9. The density of mercury is 13.5 g/mL. Determine the number of grams found in 30.0 mL of
mercury?

10. A sample weighing 30.0 grams contains 1.5 grams of salt. Determine the percent salt found in the
sample.

11. A sample contains 25% water and weighs 201 grams. Determine the grams of water in the
sample.
P a g e | 10

Experiment 1.
Metric System and Measurements
OBJECTIVES:

(a) You will become familiar with the metric system of measurements.
(b) You will make measurements of mass, length, temperature and report you answer to the correct
number of significant digits (significant figures). You will become familiar with rules for reporting the
correct number of significant digits when adding, multiplying, dividing and subtracting measurements.
(c) You will become familiar with using measuring devices (balances) and different types of
glassware.

INTRODUCTION

In chemistry, you will use two types of numbers:


a. Exact numbers and
b. Numbers that result from measurements

Exact numbers

An exact number is a number that arises when you count items or sometimes when you define a unit.
For example, there are 20 students in a class. 20 is an exact number because there are exactly 20
students not 20.00001 or 19.9999. Exact numbers have an infinite number of significant digits.

or

1 dozen = 12. Twelve in this definition is an exact number, because a dozen is exactly 12, not
11.99999 or 12.00000001 etc. but exactly 12.

Measured Numbers

A measurement is a quantity with a unit.

2 is just an exact number.


2 cm is a measurement of length. It has a number followed by a unit

In this experiment, you will make measurements using the metric system of units. The metric system
is a system of units of measurement established from its beginnings in 1874 by diplomatic treaty, to
the more modern General Conference on Weights and Measures - CGPM (Conferérence Générale
des Poids et Measures). The modern system is actually called the International System of Units or SI.
SI is abbreviated from the French Le Système International d'Unités and grew from the original metric
system. Today, most people use the name metric and SI interchangeably, with SI being the more
correct title.

SI or metric is the main system of measurement units used in science today. Each unit is considered
to be dimensionally independent from each other. These dimensions are described as the
P a g e | 11
measurements of length, mass, time, electric current, temperature, amount of a substance, and
luminous intensity.

Base SI units used in chemistry

Physical quantity Name of unit Abbreviations

Length Meter m
Mass Kilogram kg
Time Second s
Temperature Kelvin K
Amount of substance Mole mol
Electric Current Ampere A
Luminous Intensity Candela cd

In the SI system, you have the base units of measurement, as shown in the table above. Base units
in the Metric System can be converted into units that are more appropriate for the quantity being
measured by adding a prefix to the name of the base unit. These prefixes are based on multiples of
10 and when placed before the base unit makes them larger or smaller. A millimeter is one
thousandth of a meter and the symbol for a millimeter is mm and a milligram is one thousandth of a
gram and the symbol is mg.

Greek Prefixes used with SI Units and Greek Prefix and symbol for prefix

Prefix Meaning

Pico (p) one trillionth (10-12)


Nano (n) one billionth (10-9)
Micro ( ) one millionth (10-6)
Milli (m) one thousandth (10-3)
Centi (c) one hundredth (10-2)
Deci (d) one tenth (10-1)
Kilo (k) one thousand (103)
Mega (M) one million (106)
Giga (G) one billion (109)

Derived SI Units

Derived units are associated with derived quantities, for example velocity is a quantity that is derived
from the base quantities of time and length which, in SI, has the dimensions meters per second
(symbol m/s or ms-1). The dimensions of derived units can be expressed in terms of the dimensions
of the base units. In the table below are some examples of derived SI units.

Physical quantity Name of unit Abbreviation


Volume cubic meter m3
Pressure Pascal Pa
Energy Joule J
Electrical charge Coulomb C
P a g e | 12
UNCERTAINTY IN MEASUREMENTS

There is always an uncertainty in any measurement.

MEASURING VOLUME IN A GRADUATED CYLINDER

First, note that the surface of the liquid is curved. This is called the meniscus. This phenomenon is
caused by the fact that water molecules are more attracted to glass than to each other (adhesive
forces are stronger than cohesive forces). When we read the volume, we read it at the bottom of the
meniscus.

The smallest division of this graduated cylinder is 1 mL. Therefore, our reading error will be ± 0.1 mL.
An appropriate reading of the volume is 26.5 ± 0.1 mL. An equally precise value would be 26.6 mL or
26.4 mL. We have three significant figures. The "5", the last digit, is the uncertain figure.

Significant figures (significant digits) are those digits in a measured number (or result of a calculation
with measured numbers) that include all certain digits plus a final one having same uncertainty.
Anytime you make a measurement, you must report the measurement to the correct number of
significant digits, i.e. the digits you are sure of, plus the last digit, which is estimated. When you make
a measurement, you know and record the digit(s) you are sure of and a last digit, which is estimated.

However, if a measurement, made by other individuals, is reported to you, then you may be asked to
determine the number of significant digits (significant figures). Significant figures are critical when you
report scientific data because they give the reader an idea of how precisely you actually measured
your data.

Number of significant figures refers to the number of digits reported for the value of a measured or
calculated quantity, indicating the precision of the value. To count the number of significant figures in
a given measured quantity, you observe the following rules.

1. All non-zero numbers in a measurement are always significant e.g. 2.563 cm - has 4 significant
digits; 4.67546 g - has 6 significant digits, 4.8976569 mm has 8 significant figures (significant
digits).

2. Zeros in a measurement may or may not be significant depending on their position within the
measurement

(a) Zeros at the beginning of a measurement are never significant. Thus 7.34 cm, 0.734 cm
and 0.00734 cm all contain three significant figures.
P a g e | 13

(b) Zeros between two non-zero digits are always significant e.g. 2.00005 cm has 6 significant
digits; 4.03 kg has 3 significant digits, 0.008009 m has four significant digits.

(c) Terminal zeros ending at the right of the decimal point and to the right of a significant digit
are significant. Each of the following has three significant digits (figures) 2.00 cm, 2.10 cm,
60.0 cm, 0.00500 cm, 0.000707 mm.

3. Terminal zeros in which there is an implied decimal point are assumed to be not significant
e.g., 70, 700, 7000 all have one significant digit. However, if you are explicitly told that some
of the zeros are significant, they are significant. If the decimal point is explicitly shown zeros to
the left of the decimal point and after a nonzero digit are significant e.g., 90. cm has 2
significant digits; 90.0 cm has 3 significant digits. However, 900 has one significant digit.

Note: Generally, rule 3 can be resolved by using scientific notation.

Example 1. How many significant figures are present in the following numbers?

Number # Significant Figures Rule(s)

40,123 5 1, 2b

4.567 4 1

800.00 5 1, 2b, 2c

0.0004 1 1, 2a

8.1000 5 1, 2c

500.040 6 1, 2b, 2c

3,000 1 3

10.0 3 1, 2b, 2c

RULES FOR DETERMINING THE NUMBER OF SIGNIFICANT DIGITS IN CALCULATED VALUES


i.e. significant digits and calculations

1. Multiplication and division


When multiplying, or dividing measured quantities, provide as many significant figures in the
answer as there are in the measurement with the least number of significant figures.

2. Addition and subtraction


When adding, or subtracting measured quantities, provide the same number of decimal places
in the answer as there are in the measurement with the least number of decimal places.
P a g e | 15

EXAMPLES USED IN THE RULES

Last figure First figure


Number of Last figure
to be kept to be dropped
NUMBER decimal kept and/or number
(retained (dropped
places desired becomes
digit) digit)

6.422 1 6.4 6.42 6.4

6.4872 2 6.48 6.487 6.49

6.997 2 6.99 6.997 7.00

6.6500 1 6.6 6.65 6.6

7.485 2 7.48 7.485 7.48

6.755000 2 6.75 6.755 6.76

8.995 2 8.99 8.995 9.00

6.6501 1 6.6 6.65 6.7

7.4852007 2 7.48 7.485 7.49

Example 2. Give the answer to the correct number of significant figures:

1.3332 cm + 1.345 cm + 0.89 cm = ____________

Answer 2. When added on your calculator, the result that is shown is 3.5682. However, this has to
be rounded to the lowest number of decimal places (two decimal places), since we are adding
measurements. Hence the answer is 3.57 cm.

Example 3. Give the answer to the correct number of significant figures:

234 m  1.0844 m = _____________

Answer 3. 234 meters has 3 significant figures and 1.084 meters has 4 sig figs. In this calculation,
the answer must have no more than 3 significant figures.

234 m  1.084 m = 253.656 m Rounding


  254 m2.
P a g e | 16
Example 5.

Now, it is time to combine the knowledge of “uncertainty” and significant figures. The following three
examples will illustrate the importance of presenting a meaningful result after completing an
experiment.

A metallic irregular object weighs 23.56 g. The piece is dropped in a 100-mL graduated cylinder,
which contains 60.0 mL of water. The water level rises to 63.5 mL. What is the density of the metallic
object?

Answer 5. The volume of the object is 63.5 – 60.0 = 3.5 cm3 (Now this number has only 2 significant
figures, not 3). The density (d) of the metallic object is

Density = Mass/Volume

23.56
d  6.7 g / cm 3
3.5
The result has only 2 significant figures.

SCIENTIFIC NOTATION

In scientific notation, a number is written in the form A × 10n. A is a number greater than one and less
than 10 and must have the same number of significant digits as were present in the original number.
The exponent n is a positive or negative integer.

Question 4: Express the following in scientific notation:

a. 843.4 b. 0.00421 c. 845.00100

Answers: a. 8.434 × 102 b. 4.21 × 10-3 c. 8.4500100 × 102


P a g e | 17

EXPERIMENTAL PROCEDURES

PART A
Measurement of Mass

1. Weigh a 100-mL beaker on a centigram balance and record the measurement in table "A", below
(remember a centigram balance measures to ± 0.01 g). Put a circle around the estimated digit in the
measurement and determine the number of significant digits in the measurement. Note: A centigram
balance measures to two decimal places.

2. Weigh the 100-mL beaker used in part "a" using a milligram balance and record the measurement
in table "A" (remember a milligram balance measures to ± 0.001 g). Put a circle around the
estimated digit in the measurement and determine the number of significant digits in the
measurement. Note: A milligram balance measures to three decimal places.

3. Repeat part "a" and "b" above, but this time using a 125 mL Erlenmeyer flask. Record your
measurements in table A.
P a g e | 18

PART B
Determining the volume and the perimeter around face ABFE of the rectangular solid, shown
in the diagram below.

Choose one of the vertices of the cube e.g. vertex B (Three edges meet to create a vertex). Measure
the lengths of AB, BC and BF in centimeters, recording your measurements to the nearest +/- 0.01
cm in your data sheet.
Calculate the volume of the rectangular solid, showing your calculations in the space provided and
reporting your answer to the correct number of significant digits.

Calculate the perimeter around face ABFE of the rectangular solid reporting your answer to the
correct number of significant digits (perimeter means distance around face ABFE).

PART C
Calculating the density of a rectangular solid.

Your instructor will provide you with an unknown rectangular solid. Record the number or letter written
on the rectangular solid. If your rectangular solid is a pure metal, record the symbol or name of the
metal. Measure the length, width and thickness (height) of your rectangular solid in centimeters
(note: your ruler reads to ± 0.01cm). Record your measurements in table B on your data sheets.
Weigh the rectangular solid on a centigram balance and record your mass to the nearest ± 0.01 g, in
table B.
Calculate the density of your rectangular solid, reporting your answer to the correct number of
significant digits and with the correct units.
P a g e | 19

Part D
Measuring volumes of liquids.

1. Fill a 100-mL measuring cylinder to the 100-mL mark with tap water and record its volume as 100.0
mL. Approximately, half fill a test tube with water from the cylinder and record the volume of the water
left in the cylinder in table C. Remember a 100-mL measuring cylinder reads to ± 0.1 mL. Report the
number of significant digits in each of the measurements you made, in table C.

2. Select a 10-mL measuring cylinder that has 10 equal divisions per mL. Fill the 10-mL measuring
cylinder to the 10-mL mark with tap water and record its volume as 10.00 mL. Approximately, quarter
fill a test tube with water from the cylinder and record the volume of the water left in the cylinder in
table C. Remember a 10-mL measuring cylinder reads to ±0.01 mL.
Report the number of significant digits in each of the measurements you made, in the table C.
P a g e | 20

DATA SHEETS NAME: _______________________

PART A: Measurement of Mass


TABLE A

GLASSWARE Mass Number of significant digits

Mass of 100 mL beaker obtained


using a centigram balance

Mass of 100 mL beaker obtained


using a milligram balance

Mass of 125 mL Erlenmeyer flask


obtained using a centigram balance

Mass of 125 mL Erlenmeyer flask


obtained using a milligram balance
P a g e | 21

PART B
Determining the volume of a rectangular solid and perimeter around one of the faces (ABFE).

Choose one of the vertices of the cube e.g. vertex B (Three edges meet to create a vertex).

1. Length of AB in centimeters: ______________

2. Length of BC in centimeters: ______________

3. Length of BF in centimeters: ______________

4. Volume of rectangular solid: ______________

Show your calculation of the volume of the rectangular solid, in the space provided below, reporting your
answer to the correct number of significant digits.

Perimeter around Face ABFE: ______________

Show your calculation of the perimeter around face ABFE, in the space provided above, reporting your answer
to the correct number of significant digits.
P a g e | 22

PART C
Calculating the density of a rectangular solid.

Unknown number, letter or symbol of unknown: ___________________

TABLE B

Length of rectangular solid

Width of rectangular solid

Height (thickness)of rectangular solid

Mass of rectangular solid

Calculation of Density of rectangular solid

Density of Rectangular Solid = ___________________________


P a g e | 23

PART D
Measuring volumes of liquids.

TABLE C

Number of
VOLUME MEASUREMENT Volume (mL) significant digits

Volume of tap water in the P/measuring


cylinderwhen it is filled to the 100-mL mark:

Volume of tap water left in the measuring cylinder


after approximately half-filling a test tube:

Volume of tap water in the P/measuring


cylinderZhen it is filled to the 10-mL mark:

Volume of tap water left in in theP/ measuring


cylinder after approximately quarter-filling a test
tube.
P a g e | 24

POSTLAB QUESTIONS NAME: _________________


(Note that all the numbers are measured numbers)

1. How many significant digits are there in each of the following measurements?

(a) 2688 cm (b) 3.507 g

(c) 5.700 km (d) 0.00400 kg

(e) 24.00300 m (f) 2400 m

(g) 9.5000 x 10-8 m (h) 0.0020056 g

2. Round each of the following measurements to two significant digits.

(a) 0.00258 cm (b) 28.23 g

(c) 5.998 cm (d) 1.75 g

(e) 1.05 g

3. Do the following calculations and round your answer to the correct number of significant digits.

(a) 16.5 cm + 8 cm + 4.37 cm = (b) 13.57 cm - 6.3 cm =

(c) 672 cm × 39.864 cm × 10 cm = (e) 13. 57 cm / 6.3 cm2 =

(f) (14.86 cm + 13.7 cm) × (65.346 cm - 4.10 cm) =


(43.888 cm - 32.888 cm)

4. Write each of the following measurements using scientific notation.

(a) 0.000450300 m

(b) 4567.67 g

5. Why are significant digits important when taking data in the laboratory?
P a g e | 25
6. What is wrong asking about the number of significant digits in 4.56?

7. Indicate how many significant figures there are in each of the following measured values.

a. 246.89 = __________

b. 107.856 = __________

c. 100.2 = __________

d. 0.646 = __________

e. 1.006 = __________

f. 0.005640 = __________

g. 14.600 = __________

h. 0.0002 = __________

i. 800000 = __________

j. 350.670 = __________

k. 1.0000 = __________

l. 320001 = __________

8. Calculate the answers to the appropriate number of significant figures.

a. 32.456 + 135.0 + 1.2345 = __________

b. 246.14 + 238.234 + 98.1 = __________

c. 658.0 + 23.34567 + 1345.29 = __________


P a g e | 26
9. Calculate the answers to the appropriate number of significant figures.

a. 24.7 × 1.8 = __________

b. 45.65 × 0.25 = __________

c. 81.04 g × 0.010 = __________

d. 6.47 × 63.7 = __________

3.678
e.  __________
64.1

0.234
f. 1.678   __________
0.42

g. (2.67×1012) × (3.7 × 104) = __________

1.2  10 3
h.  __________
2.4  10 4

10. Round off the numbers to the indicated significant figures.

a. 230 to one sig fig __________

b. 2345 to three sig fig __________

c. 789000 to two sig fig __________

d. 6700 to one sig fig __________

11. Convert the following numbers to scientific notation using significant figures.

a. 2340000000 __________

b. 0.00034 __________

c. 123000000 __________

d. 0.000160 __________
P a g e | 27

Experiment 2.
Physical and Chemical Changes
Objectives

Learning Objective 1: To observe physical and chemical changes.


Learning Objective 2. To identify physical and chemical changes.

Introduction

A physical change is one in which the appearance of a substance changes but its composition or
identity is unaffected. No new substances are formed. Physical changes are usually reversible.
Examples of physical changes include the boiling of water to produce steam, the filing of a metal to
produce dust or filings, and the dissolving of table salt in water. In most cases, one or two simple
processes are all that is needed to reverse the physical change. For example, salt that has been
dissolved in water may be recovered water by evaporating the water from the solution.

A chemical change (Chemical reaction) is one that results in a change in the composition and
identity of a substance. New substances are formed. Chemical changes are usually irreversible.
Examples of chemical reactions include the burning of wood to form carbon dioxide and water, the
rusting of iron to form iron oxide, and the heating of limestone to produce lime and carbon dioxide.
Usually, reversing a chemical reaction requires one or more chemical reactions.

In a chemical reaction, one or more substances called reactants are necessary. The newly formed
substance(s) is(are) called the product(s) and these product(s) always differ from the reactants in
their physical and chemical properties. Physical properties are properties that do not involve the
reaction of a substance with other substances. They include such things as color, state (solid, liquid,
or gas), density, and melting and boiling points. Chemical properties are displayed when the
substance reacts to produce products or undergoes a chemical change. A chemical reaction is
indicated by any of the following observations.

1. Change of color.
2. Production of heat, light, or sound.
3. Evolution of gas.
4. Formation of a precipitate.

These observations are not infallible indications that a chemical reaction has occurred. For example,
when ice forms from liquid water, a solid is present where none was before, but a physical change
rather than a chemical reaction has occurred. In this case, one would classify the change as physical
based on the ease of reversing it.

SAFETY INFORMATION:

1. NO EATING or DRINKING IS ALLOWED IN CHEMISTRY LABORATORIES.


2. OBTAIN and WEAR EYE GOGGLES AT ALL TIMES.
3. NOTIFY YOUR INSTRUCTOR IF YOU HAVE ANY QUESTIONS.
4. WASH YOUR HANDS BEFORE LEAVING THE LABORATORY.
P a g e | 28
Special Note: Quantities need not be measured precisely in this experiment. If reagents are
dispensed from dropper bottles, one squirt from the dropper should deliver about 1 mL (or
approximately 20 drops), or 1 cc, of the reagent. For reagents dispensed from bottles with stoppers or
screw caps, simply estimate the amount of liquid as you pour it into a test tube. An ordinary large test
tube holds about 20 mL. Therefore, liquid quantities can be estimated as follows:

Quantity of Liquid (Approximate height in a large test, 20-mL, tube):

10 mL is about 1/2 full


5 mL is about 1/4 full
2 mL is about ½ inch
1 mL is about 20 drops

Whenever laboratory experiments are qualitative rather than quantitative in nature, volumes of liquid
can be estimated in this manner.

For all demonstrations and tests, you must record in your data sheet the appearance of the
substances before the change and what occurred during and after the change(s).

Procedure C. Three Pennies Experiment

NOTE: Very clean pennies are required.

Procedure D. Sublimation of Iodine crystals

NOTE: Hot plate gives satisfactory results.

STUDENT EXPRIMENTS:

NOTE: Tests 1, 6, and 10 can be run effectively in a spot plate.

MATERIALS: Steel wool, well plates, small test tubes, calcium carbonate, baking soda,
wooden splints, magnesium ribbon, potassium chlorate, 0.1 M NaCl, 0.1 M AgNO3, CaCl2
NH4NO3, univeral indicator, 0.5 M CuCl2, Dry ice, 6 M NaOH, zinc powder.
P a g e | 29

PRELAB QUESTIONS NAME: ___________________

1. Identify the three Physical States of matter.

2. Identify four changes which are indicative of a chemical change.

3. Provide the two examples of chemical changes discussed in the experimental handout.

4. Fill in the blanks.

a. 1 mL is about ______________

b. 1cc is about ______________

c. ½ full test tube is about ______________

5. Circle the piece of equipment which can be used to more precisely determine the volume.

Dropper Test Tube


P a g e | 30

INSTRUCTOR DEMONSTRATIONS

A. Baking Soda and Vinegar

What will happen if I mix vinegar and baking soda? Is it a chemical change or a physical
change?

Procedure:

1. Weigh about 5 grams of baking soda using a spatula. Place the baking soda in into a container.
2. Pour about 2 mL of vinegar into the container.
3. Note observations.

B. Clock Reaction

Color changes as a function of time, when with mixing solutions - Is it a chemical change or a
physical change?

Procedure:

NOTE: The following solutions are prepared, in advance, by the laboratory coordinator.

Solution A: 250 mL of 3% Hydrogen peroxide (H2O2)

Solution B: 7.25 g Potassium Iodate (KIO3) in 100 mL of water. Add 2.15 mL of 6.0 M sulfuric acid
(H2SO4). Warm and stir until KIO3 dissolves. Dilute to 125 mL with water.

Solution C: 2.6 g malonic acid and 0.55 g Manganese (II) sulfate (MnSO4.H2O) in 100.0 mL water.
Heat 50 mL of water to a boil in a 100-mL beaker. In a 50-mL beaker, make a slurry of 0.2 g starch in
5.0 mL deionized water. Pour the slurry into the boiling water. Stir to dissolve. Now pour 15 mL of this
into the malonic acid/manganese(II) sulfate solution. Dilute to 125 mL with water.

Combine solution A and B. Add a magnetic stirring bar and stir to create a large vortex in the mixture.
Now pour in solution C. Use cell phones timers, note seconds for reaction.
P a g e | 31
C. Three Pennies Experiment

Has the Alchemists' Dream Come True? Is it a chemical change or a physical change?

The Alchemists' dream was to change base metals such as lead into gold. In this experiment, you
start with three clean shiny copper pennies (minted on or before 1982) and end with one copper
penny (base metal left unchanged), one silver penny (coated with zinc), and one golden penny
(bronzed penny). Your problem is to classify the process which changes the copper penny to the
silver penny and the silver penny to the gold penny as a physical or chemical process.

Leave one of the three pennies on the bench top (this penny remains unchanged during the
experiment). Place a pea size scoop of granular zinc metal in a 50-mL beaker. Put about 20 mL of
2.0 M ZnCl2 in the beaker, drop two pennies in the solution and heat on a hot plate. After the solution
boils for about 5 minutes and the top side of the pennies look silvery turn the pennies over using your
spatula or crucible tongs. Continue heating for another 5 minutes or until both sides of the pennies
are silver in color. Turn off the hot plate and take the beaker off the hot plate. Let the solution cool
and carefully pour the solution into the waste container. Remove the pennies and dry them. Place the
solid zinc in the appropriate waste container. What color are the pennies?

Put 25 mL of cold water in the 50-mL beaker. Light a Bunsen burner and adjust the flame so that it is
not too hot (no inner light blue cone but not a yellow flame either). Pick up one of the pennies with
your crucible tongs and carefully heat it in the top of the flame until it suddenly turns golden.
Immediately remove the penny from the flame, and drop it in the beaker of cold water. (If you heat the
penny too long or with too hot a flame, the penny will melt.) What color is the penny?

D. Sublimation of Dry Ice.

Procedure:

Obtain dry ice from the prep room and place it on a workbench. Note your observations.
DO NOT TOUCH the dry ice. It is -80oC. It will burn your hand. Use CAUTION.
P a g e | 32

STUDENTS EXPERIMENTS
Test 1: Copper(II) sulfate (CuSO4) solution and steel wool

a. Place about 10 mL of the blue copper(II) sulfate solution in a small beaker.

b. Dip a piece of steel wool, which is composed mainly of elemental iron, into the blue copper(II)
sulfate solution by holding it in the solution with tongs for about 30 seconds and then take it out.

c. Examine the coating on the steel wool. Record your observations. Now put the steel wool back into
the solution and let it remain for at least 15 minutes while you perform the other procedures.

d. Record your observations.

Test 2: Decomposition of the calcium carbonate (CaCO3).

a. Working in the fume-hood, add dilute hydrochloric acid (HCl) dropwise to a test tube containing a
pea size portion of calcium carbonate.

b. Perform a splint test on any gases that are produced: light a wooden splint and bring close to the
mouth of the test tube.

c. Record your observations.

Test 3: Burning of the magnesium ribbon.

a. Working in the fume-hood, ignite a small strip of magnesium ribbon by holding it with tongs in the
flame of the burner. Do not look directly at the burning magnesium flame.

b. Record your observations.

Test 4: Decomposition of potassium chlorate (KClO3).

a. Mix a small quantity of solid potassium chlorate with solid manganese(IV) oxide, MnO2, in a test
tube and attach the test tube to a stand with a clamp at a 45º angle.

b. Working in the fume-hood, heat the contents of the test tube over the Bunsen burner flame.

c. Perform a splint test on any gases that are produced: light a wooden splint and bring it close to the
mouth of the test tube.

d. Record your observations. (Note: MnO2 is a catalyst in this reaction and so does not need to be
part of the chemical equation. A catalyst is a reagent that speeds up the chemical reaction without
being consumed).
P a g e | 33
Test 5: Thermal decomposition of sodium bicarbonate (NaHCO3).

a. Place a small quantity of solid sodium bicarbonate in a test tube.

b. Attach the test tube to a stand with a clamp at a 45º angle.

c. Working in the fume-hood, heat the contents of the test tube using a Bunsen burner flame.

d. Perform a splint test on any gases that are produced.

e. Record your observations.

Test 6: Precipitating sodium chloride solution (NaCl) with a silver nitrate (AgNO3) solution

a. Place 10 drops of sodium chloride solution in a test tube.

b. Add 2 drops of silver nitrate solution to the test tube.

c. Record your observations.

Test 7: Formation of the calcium chloride (CaCl2) solution.

a. Put about a teaspoon of calcium chloride (CaCl2, a solid) in a test tube.

b. Touch the bottom of the test tube to the inside of your arm to get a sense of its temperature.

c. Add deionized water to the test tube (about 1/3 full) and stir with a glass stirring rod.

d. Touch the bottom of the test tube to the inside of your arm again. If you don’t feel a difference, add
more solid calcium chloride. Record your observations.

e. When finished, place the contents of the test tube in the waste container. Rinse the test tube with
water. You do not need to dry the test tube.
P a g e | 34

Test 8: Formation of an ammonium nitrate (NH4NO3) solution.

a. Put about a teaspoon of ammonium nitrate (NH4NO3, a solid) in a test tube.

b. Touch the bottom of the test tube to the inside of your arm to get a sense of its temperature.

c. Add deionized water to the test tube (about 1/3 full) and stir with a glass stirring rod.

d. Touch the bottom of the test tube to the inside of your arm again. If you don’t feel a difference, add
more potassium nitrate. Record your observations.

e. When finished, place the contents of the test tube in the waste container. Rinse the test tube with
water. You do not need to dry the test tube.

Test 9: A natural indicator (red cabbage juice is "universal indicator").

a. Obtain 3 test tubes and fill each one with 5-10 drops of universal indicator.

b. To the first test tube, add an equal amount of deionized water.

c. To the second test tube, add an equal amount of 0.10 M HCl (hydrochloric acid).

d. To the third test tube, add an equal amount of 0.10 M NaHCO3 (baking soda).

e. Note any color changes and record them on your data sheet.

f. Pour the liquids into a waste container. Thoroughly rinse the test tubes with water.

Test 10: Magnesium (Mg) with copper(II) chloride (CuCl2) solution.

a. Add about 2-3 mL of CuCl2 (copper(II) chloride) solution to a test tube.

b. Record your observations. (Include color, temperature, states of matter, etc.)

c. To this test tube, add a 1 inch piece of magnesium ribbon.

d. Touch the test tube to the inside of your arm. Wait for a minute and record your observations.
(Include color, temperature, states of matter, etc.)

e. When finished, place the magnesium and copper chloride solution in a labeled waste container.
Rinse the test tube with water (it does not need to be dry).
P a g e | 35
DATA SHEETS

For each reaction, write down your observation(s) before and after the reaction that you believe
provide evidence that a chemical change took place. Be clear and concise; use only enough detail to
communicate your findings.

TYPE OF
EXPERIMENT OBSERVATIONS CHANGE
(Physical or
BEFORE CHANGE AFTER CHANGE Chemical)

Test #1

Test #2

Test #3

Test #4

Test #5

Test #6
P a g e | 36

Test #7

Test # 8

Test #9

Test #10
P a g e | 37

POST LABORATORY ASSIGNMENT NAME: ________________

Label each process as a physical (P) or chemical (C) change:

a. reflection of light on a mirror ( )

b. bending a copper wire ( )

c. grinding spices ( )

d. burning paper ( )

e. slicing potatoes for fries ( )

f. mixing sugar with water ( )

g. strongly heating a mixture of steel wool and sulfur ( )

h. a nail rusting ( )

i. paper ripping ( )

j. wood burning ( )

k. mixing water and food coloring ( )

l. food molding (rotting) ( )

m. tarnishing copper ( )

n. dyeing fabric ( )
P a g e | 39

Experiment 3.
Density Determination
LEARNING OBJECTIVES
The students will be able to learn that:

1. The density is an intensive property and it is independent of the amount of sample used.
2. The volume of an object can be determined by the volume of the water it can displace.
3. The density of a mixture is between the densities of the constituents forming the mixture.

BACKGROUND
Density is a physical property of a substance that is often useful part of its identification. Density of
a substance is described as the mass of a unit volume and it is an intensive property. Intensive
properties do NOT depend on the amount of the sample used. Density may be thought of as how
heavy an object is for its size, or the ratio of an object’s mass to its volume.

mass m (g)
Density   d= Equation 1.
volume V ( mL)

The units of density of a solid object are g/cm3 and for the liquids are in g/mL. (1 cm3 ≈ 1 mL).

There are three parts to this experiment.

a. Density of the regularly shaped objects.


b. Density of the irregularly shaped objects (water displacement method).
c. Density of a mixture

In these investigations, you will determine the density of several objects by measuring their mass and
volume. The density of a sample usually varies inversely with temperature. The density of an object
usually decreases with increasing temperature. Therefore, when reporting the density of a sample,
you must report the temperature in the laboratory during the experiment; i.e. “the density of water is
0.997 g/mL at 25 oC”.
P a g e | 40

SAFETY INFORMATION
1. NO EATING or DRINKING IS ALLOWED IN CHEMISTRY LABORATORIES.
2. OBTAIN and WEAR EYE GOGGLES AT ALL TIME.
3. NOTIFY YOUR INSTRUCTOR IF YOU HAVE ANY QUESTIONS.
4. WASH YOUR HANDS BEFORE LEAVING THE LABORATORY.

MATERIALS
10-mL graduated cylinder, 100-mL graduated cylinder, centigram balance, metric ruler, rectangular or
cubic shaped objects (PVC, Wood, Brass, Iron, Copper, Aluminum), marble chips, unknown salt
solutions.

DRY LAB SIMULATION: http://phet.colorado.edu/sims/density-and-buoyancy/density_en.html


P a g e | 41

PRELAB QUESTIONS NAME: ___________________

1. Identify the formula for determining the density.

2. List the units for the variables found in the density formula.

3. Identify the formula for a rectangular solid (cuboid).

4. Look at the table of densities found on page 43 in the lab book. Based on the information provided
predict if rubber will float.

5. Look at the table of densities found on page 43 in the lab book. Identify one material which will
float.
P a g e | 42

PART A: DENSITY OF A REGULARLY SHAPED OBJECT

h
l
w
Figure 1: Determination of the volume of a cubic or rectangular shaped object

The volume of the object (cm3) = length (cm) × height (cm) × width (cm) Equation 2.

PROCEDURE: SAMPLE #: ____________

1. Obtain a cubic/rectangular object from the box and record the sample number.

2. Weigh the object using a centigram balance.

3. Measure the length, the width, and the height of the object.

4. Calculate the volume of the object using the formula given in Equation 2.

5. Calculate the density of the object using Equation 1.

6. Repeat the experiment and take the average value of the density. This is needed to minimize
error in the experiment.

NOTE: ALL MEASUREMENTS WILL HAVE TWO DIGITS AFTER THEIR DECIMAL POINTS.
P a g e | 43
DATE: ______________ NAME: ___________________________

DATA SHEET FOR PART A. Trial 1 Trial 2

1. Measure and record the temperature. ____________ oC _________ oC.

2. Mass of the object. ____________ grams. _________ grams.

3. Length of the object ____________ cm. _________ cm.

4. Width of the object ____________ cm. _________ cm.

5. Height of the object ____________ cm. _________ cm.

6. Volume of the object: V = L × H × W ____________ cm3. _________ cm.

7. Density of the object (line (2) / line (6)) ____________ g/cm3 _________ g/cm3.

8. The average density of the object: ___________ g/cm3 at _________ oC.

9. Find the accepted density of the sample from the table provided below and calculate the percent
error in your determination:

MATERIAL Density (g/cm3) at 20oC


Acrylic 1.18
Aluminum 2.73
Brass 8.55
Wood 0.40
Copper 8.90
Gold 19.3
Iron 7.68
Lead 11.4
Marble Chips 2.70
Nylon 1.30
PVC 1.41
Graphite 2.16
Steel 7.85
Water 0.99

Table 1: Density of some common materials.

Accepted value  exp erimental value


Percent Error  | |  100 = _______________
Accepted value
P a g e | 44

Part B: Determination of the density of an irregular shaped objects


For each trial, pour about 20 – 30 mL of water into a 50-mL graduated cylinder. Accurately record the
volume of water to the nearest 0.1 mL. Dry the outside of the cylinder with a paper towel. Obtain the
mass of the graduated cylinder and water with your balance and record the mass to the nearest 0.01
g. Weigh 20 marble chips (for the first trial) and 30 marble chips (for the second trial). Carefully drop
the chips in the graduated cylinder and gently tap out any air bubbles. Record the new volume level
in the graduated cylinder to the nearest 0.1 mL. Calculate the density of the marble chips to the
nearest 0.1 g/mL.

Figure 2: Reading the volumes from graduated cylinders.

Note: Segregate the large marble chips and do not use them.
P a g e | 45

DATE: ______________ NAME: ___________________________

DATA SHEET FOR PART B.

Trial 1 (20 chips) Trial 2 (30 chips)

1. Mass of objects: ________ g ________ g

2. Starting volume of water in the graduated cylinder: ________ mL ________ mL

3. Ending volume of water in the graduated cylinder: ________ mL ________ mL

4. Volume of objects: (Line 3) – (Line 2) ________ mL ________ mL

5. Density of the marble chips: (Line 1) / Line 4) ________ g/mL ________ g/mL

Calculate the average density of the marble chips (show your work): ________ g/mL.

6. The accepted density of the sample (marble chips) is 2.70 g/mL. Calculate the percent error
in your determination:

Accepted value  exp erimental value = __________ %


Percent Error  | |  100
accepted value
P a g e | 46

PART C: DETERMINATION OF DENSITY OF SALT-WATER MIXTURE


Obtain an unknown saline solution from your instructor.
Record the number of the unknown sample __________ .

PROCEDURE and DATA SHEET FOR PART C. Trial 1 Trial 2

1. Weigh an empty 10-mL graduated cylinder: ________ g _______ g

2. Fill the graduated cylinder with the unknown sample to exactly 10-mL mark. V = 10.00 mL.

3. Weigh the 10-mL graduated cylinder and the sample again: ________ g _______ g

4. Mass of the sample (Line 3) – (Line 1): ________ g _______ g

5. Density of the unknown sample: (Line 4) / (Line 2) ________ g/mL. ______ g/mL

6. The average density of the unknown sample: __________ g/mL

7. Determine the percent composition of your sample from the chart below using the average density
of the two trials:

8. Measure and record the temperature. _________ oC.

Salt (%) Density (20oC)

(Pure water) 0.0 0.998 g/mL


5.0 1.036 g/mL
10.0 1.074 g/mL
15.0 1.112 g/mL
20.0 1.152 g/mL
25.0 1.193 g/mL
(Pure salt) 100.0 2.165 g/mL

9. The unknown sample ___________ has a density of ____________ g/mL. The concentration of

the salt is ____________ %.


P a g e | 47

POST-LABORATORY QUESTIONS NAME: __________________


(Observe the significant figures rules in your calculations).

1. What is the density of a piece of concrete that has a mass of 8.76 g and a volume of 3.07 cm 3?

2. Cobalt has a density of 8.90 g/mL. What volume would 17.8 g of cobalt occupy?

3. Limestone has a density of 2.72 g/cm3. What is the mass of 24.9 cm3 of limestone?

4. A rectangular object has the following dimensions: length 2.00 cm, height 3.00 cm and the width
1.50 cm. It weight is 34.26 grams. What is the density of the object?
P a g e | 48
5. A cylindrical metallic piece weighs 20.00 grams. When it is dropped into a graduated cylinder, the
volume of water rises from 50.0 mL to 55.5 mL. What is the density of the piece?

6. The density of alcohol is 0.80 g/mL and that of the water is 1.00 g/mL. What is the density of a
mixture prepared by mixing 50.0 mL of water and 50.0 mL of alcohol assuming that the volumes are
additive?
P a g e | 49
7a. The density of sugar solutions at various concentrations is tabulated below. Using the chart
construct the graph density (y-axis) versus % weight (x-axis). Note: Must use Excel® to graph the
below data.

Density % by
(g/mL) weight (w/v)
0.998 0
1.018 5
1.038 10
1.059 15
1.081 20
1.104 25
1.127 30
1.151 35
1.176 40

7b. (OPTIONAL). A student determines the density of degassed (flat soda) Sprite®. The density is
found to be 1.120 g/mL. Using the graph you constructed in 6a, determine the sugar content in a
12-oz can of Sprite®. (12 fl oz can equals 335 mL). (Hint: % weight by volume is the mass of sugar
in 100 mL of drink, e.g. if the density is 1.018 g/mL there are 5 grams of sugar in 100 mL of soda).
P a g e | 51

Experiment 4.
Specific Heat of a Metal
LEARNING OBJECTIVES
The students will be able to learn

1. The concept of conservation of energy.


2. That heat flows from a hot object to a cold one.
3. That metals are good conductors of heat. Metals have low specific heat; for a given energy input to
a given mass, the temperature rise of metals will surpass the temperature rise of non-metals.

BACKGROUND
The First Law of Thermodynamics states that energy can neither be created nor destroyed; it can
only be transformed to a different form of energy, e.g. in batteries, the chemical energy is converted
into electrical energy and in automobile engines, where the gasoline is burned, the chemical energy is
converted into kinetic energy. Basically, the energy of the Universe is a constant. The Universe is
composed of a System (in this experiment, the calorimeter) and its Surroundings (everything else).

Heat energy is measured in terms of calories or Joules. One calorie is the amount of heat energy
required to increase the temperature of 1.00 gram of water by 1.00 ᵒC. Specific Heat is a physical
property of a substance. It is defined as “the amount of heat energy required to increase the
temperature of 1.00 g of any substance by 1.00 ᵒC. Therefore, substances with low specific heat
require less heat energy to raise their temperatures.

4.18 joules
Specific heat of water, c 
1.00 g  1.00 o C

The unit of the specific heat is J/(g×ᵒC).

There are two parts in this experiment:

a. Determination of the specific heat of metallic copper.


b. Determination of the specific heat of an unknown metal.

In Part A of these investigations, you will determine the specific heat of metallic copper. Then, in Part
B, you will determine the specific heat of an unknown sample and identify it by using the specific heat
value provided in the Table 1.
You will use a calorimeter to determine the specific heat of the metal. The calorimeter (in Greek calor
means heat) is a device that measures the specific heat of a metal using the temperature changes of
the metal and the water (see Figure 1).
P a g e | 52
The amount of heat energy needed to increase the temperature of an object is given by the following
formula:
Q = m × c × ΔT Eq. 1

Q = heat energy (Joules)


m = mass of the metal (g)
c = specific heat of the metal J/(g.ᵒC)
ΔT = change in temperature (ᵒC)

Assuming an ideal calorimeter, no heat escapes from the cup (the heat is not lost to the
surroundings) and the heat is exchanged between the hot metal and the cool water:

DRY LAB SIMULATION: http://www.chemfiles.com/flash/calorimeter.swf


P a g e | 53
Now, using the First Law of Thermodynamics, we will write the following expression:

Heat lost by the hot metallic object = Heat gained by the water

In this experiment, the metal will be heated in a test tube immersed in boiling water that is contained
in a beaker. Then, the metal will be dropped into the calorimeter (doubled Styrofoam cups originally
containing water at room temperature).

Qlost by the metal = Qgained by the water Eq. 2

mmetal × cmetal × (Tmetal – Tfinal) = mwater × cwater × (Tfinal – Twater) Eq. 3

where

mmetal = mass of the metal used (grams)


cmetal = specific heat of the metal (for copper c = 0.38 J/(g×ᵒC)
Tmetal = temperature of the boiling water (100ᵒC)
mwater = mass of the water in the calorimeter (grams)
cwater = specific heat of water; c = 4.18 J/(g×ᵒC)
Twater = the initial temperature of the water in the calorimeter (ᵒC)
Tfinal = the temperature of the water and the metal in the calorimeter (ᵒC)

PRE LABORATORY WORKED EXAMPLE

Example. A 50.00 g sample of copper at 99.0ᵒC is dropped into a calorimeter containing 170.00 g of
water at 22.0 ᵒC, producing a maximum resulting temperature of 24.0ᵒC. Calculate the specific heat
of the copper sample.

Solution: Qlost by the metal = Qgained by the water

mmetal × cmetal × (Tmetal – Tfinal) = mwater × cwater × (Tfinal – Twater)

(50.00 g) × (ccopper) × (99.0 - 24.0 ᵒC) = (170.00 g) × (4.18)(J/g×ᵒC) × (24.0 – 22.0 ᵒC)

3750 × ccopper = 1421.2 J

ccopper = 0.38 J/(g×ᵒC)


P a g e | 54

MATERIALS
100-mL graduated cylinder, 250-mL and 400-mL beakers, weighing dish, large test tube, heater-
stirrer, utility clamp, ring stands, Styrofoam coffee cups, thermocouples (2), Labquest ®, centigram
balance, deionized water, copper shots, unknown metals, and boiling chips.

SAFETY INFORMATION
1. NO EATING or DRINKING IS ALLOWED IN CHEMISTRY LABORATORIES.
2. OBTAIN and WEAR EYE GOOGLES AT ALL TIME.
3. NOTIFY YOUR INSTRUCTOR IF YOU HAVE ANY QUESTIONS OR ALLERGIES TO THE
METALS.
4. CLEAN UP WASH YOUR HANDS BEFORE LEAVING THE LABORATORY.

Note: A thermometer can be used instead of a thermocouple.


P a g e | 55

PRELAB QUESTIONS NAME: ____________________

1. Identify the specific heat equation. Provide the names and identify the units of all the variables.

2. An unknown metal is dropped into 127 grams of water. The temperature of the water has been
raised from 25OC to 28OC. Using the specific heat of water found on page 57 determine the amount
of heat gained by the water.

3. A 20.0-gram piece of metal at 88OC is cooled by a sample of water to 25OC. The amount of
energy (energy given off by the metal is = energy absorbed by the water) is 340 Joules. Determine
the specific heat of the unknown metal.

4. Identify the material based upon the specific heat calculated and the published values found within
Table 1 of this experiment.
P a g e | 56

PART A: Specific heat of metallic copper

PROCEDURE:

1. Obtain a ring stand, a heater, and a utility clamp and assemble them according to Figure 2.

2. Obtain a 400-mL beaker, drop a couple of boiling chips in it, and fill it with about 250 mL of tap
water.

3. Place the 400- mL beaker on the heater. Start heating the water.

4. Obtain a test tube and place about 50 g of copper pieces in it. Place one of the thermocouples in
the test tube.

5. Insert the test tube, suspended by the utility clamp, into the water and continue to heat the water
until the water starts to boil.

6. Continue to boil the water while heating the metal. Record the temperature when it reaches a
constant value (about 98 to 100ᵒC).

7. Assemble the Styrofoam cups, the beaker, cardboard cover, thermocouple, and the Labquest ®
according to the Figure 1.
P a g e | 57
8. Using a 100-mL graduated cylinder, measure 100.0 mL of deionized water, and place it in the
calorimeter. This equals to 100.0 g of water since the density of water is 1 g/mL. Measure the
temperature (from LabQuest, first thermocouple) of the water in the calorimeter and record it.

9. Read the temperature of the metal sample (from LabQuest, second thermocouple), record it, and
using the utility clamp as a test tube holder, rapidly transfer the sample into the calorimeter and stir
the contents of the calorimeter with the thermocouple (first thermocouple).

10. Observe the temperature rise and record the highest temperature. Your instructor will guide you
on how to setup the time-based data acquisition using the LabQuest®.

11. Empty and dry the calorimeter with a paper towel. Replace the water, weigh another sample
(make sure it is dry), and do another trial. Record the maximum temperature. Compute the average
value.

12. Empty the calorimeter again. Dry the sample with a paper towel, and return the sample to the
designated container.

MATERIAL Specific heat J/(g×ᵒC)


Aluminum 0.900
Brass 0.380
Copper 0.387
Silver 0.234
Gold 0.129
Iron 0.448
PVC 1.34
Marble Chips 0.860
Tin 0.210
Sand 0.830
Steel 0.120
Water 4.18

Table 1: Specific heat of some common materials.


P a g e | 58

PART B: Specific heat of unknown metal

PROCEDURE:
1. Obtain a ring stand, a heater, and a utility clamp and assemble them according to Figure 2.

2. Obtain a 400-mL beaker, drop a couple of boiling chips in it, and fill it with about 250 mL of tap
water.

3. Place the 400- mL beaker on the heater. Start heating the water.

4. Obtain a test tube and place about 50 grams of unknown metal pieces in it. Place one of the
thermocouples in the test tube.

5. Insert the test tube, suspended by a utility clamp, into the water and continue to heat the water until
the water starts to boil.

6. Continue to boil the water while heating the metal. Record the temperature when it reaches a
constant value (about 98 to 100 ᵒC).

7. Assemble the Styrofoam cups, the beaker, cardboard cover, thermocouple, and the Labquest®
according to Figure 1.

8. Using 100-mL graduated cylinder, place exactly 100.00 mL (assume a density of 1 g/1 mL) of
deionized water in the calorimeter. Measure the temperature (from LabQuest) of the water in the
calorimeter and record it.

9. Read the temperature of the metal sample (from LabQuest, second thermocouple), record it, and
using the utility clamp as a test tube holder, rapidly transfer the sample into the calorimeter and stir
the contents of the calorimeter with the thermocouple (first thermocouple).

10. Observe the temperature rise and record the highest temperature. Your instructor will guide you
on how to set up the time-based data acquisition using the LabQuest®.

11. Empty and dry the calorimeter with a paper towel. Replace the water, weigh another sample
(make sure it is dry), and do another trial. Record the maximum temperature and compute the
average reading.

12. Empty the calorimeter again. Dry the sample with a paper towel, and return the sample to the
designated container.

NOTE: ALL MEASUREMENTS WILL HAVE TWO DIGITS AFTER THEIR DECIMAL POINTS.
P a g e | 59
DATE: _____________ NAME: ____________________
DATA TABLE FOR SPECIFIC HEAT OF COPPER

Known Object: COPPER

Trial 1 Trial 2

1. mass of the weighing dish ____________ g _________ g

2. mass of weighing dish + copper ____________ g _________ g

3. mass of copper: (line 2) – (line 1) ____________ g _________ g

4. mass of water in the calorimeter ____________ g _________ g

6. temperature of metal ____________ ᵒC _________ ᵒC

7. initial temperature of water ____________ ᵒC _________ ᵒC

8. final temperature of water ____________ ᵒC _________ ᵒC


(in the calorimeter)

9. heat gained by water ___________ J _________ J


[(100) × (4.18) × (line 8) – (line 7)]

10. heat lost by the copper# ___________ J _________ J


[-(line 9)]

11. specific heat of copper ___________ J/(g×ᵒC) _________ J/(g×ᵒC)


(solve for c using (line 10) / [(line 3) x ((line 8) – (line 6))]

12. average specific heat of copper _________ J/(g×ᵒC)

# Denotes heat lost is negative.


P a g e | 60

DATE: _____________ NAME: ____________________

DATA TABLE FOR SPECIFIC HEAT OF UNKNOWN METAL

Unknown Object: ____________

Trial 1 Trial 2

1. mass of the weighing dish ____________ g _________ g

2. mass of weighing dish + metal ____________ g _________ g

3. mass of metal: (Line 2) – (Line 1 ____________ g _________ g

4. mass of water in the calorimeter ____________ g _________ g

6. temperature of metal ____________ ᵒC _________ ᵒC

7. initial temperature of water ____________ ᵒC _________ ᵒC

8. final temperature of water ____________ ᵒC _________ ᵒC


(in the calorimeter)

9. heat gained by water ___________ J _________ J


[(100) × (4.18) × (Line 8) – (Line 7)]

10. heat lost by the metal# ___________ J _________ J


[-(Line 9)] EHFDXVHWKHWHPSHUDWXUHFKDQJHLVQHJDWLYH

11. Temperature change of the metal ____________ ᵒC ____________ ᵒC


Line(8) - Line(6)

12. specific heat of metal ___________ J/(g×ᵒC) _________ J/(g×ᵒC)


solve for c using line (10) / [Line (3) x Line (11)]

13. average specific heat of unknown metal _________ J/(g×ᵒC)

# Denotes heat lost is negative.


P a g e | 61

POST-LAB QUESTIONS NAME: ___________________


(Observe the significant figures rules in your calculations).

1. The specific heat of water is 4.18 J/(g×ᵒC). How much heat energy is required to raise the
temperature of 70.00 g of water by 20.00ᵒC?

2. How much is heat required to raise the temperature of 50.00 grams of copper from 25.0ᵒC to 45.0ᵒC?
The specific heat of copper is 0.387 J/(g×ᵒC).

3. For a given energy input, which material will have the greater temperature rise: a sample of 10.0 g
of water or a sample of 10.0 grams of copper? Why?
P a g e | 62

ADDITIONAL POST-LAB QUESTIONS.


4. A 150.0 g sample of aluminum, c = 0.90 J/(g×ᵒC), is heated to 250.0ᵒC and rapidly dropped into a
calorimeter containing 100.0 g of water, c = 4.18 J/(g×ᵒC), at 22.0ᵒC. What is the final temperature of
water in the calorimeter? Assume no heat escaped from the calorimeter.

5. A 50.0 g sample of zinc is heated, then placed in a calorimeter containing 100.0 g of water. The
temperature of water increases from 22.00oC to 24.50oC. The specific heat of zinc is 0.390
J/(g×oC). What was the initial temperature of the zinc metal sample? (final temperatures of zinc and
water are the same and no heat escapes from the calorimeter).

6. A 15.0 g sample of gold is heated, then placed in a calorimeter containing 100.0 grams of water. The
temperature of water increases from 29.0oC to 32.0oC. The specific heat of gold is 0.130 J/(g×oC).
What was the initial temperature of the gold metal sample? (Assume no escape from the calorimeter).

7. A 30.0 g sample of metal is heated to 109.0oC, then is placed in a calorimeter containing 50.0 g of
water. The temperature of water increases from 21.0oC to 23.0oC. What is the specific heat of
sample?
P a g e | 63

Experiment 5.
Flame Test and Chemical Fingerprinting
Learning Objectives:
1. To study the characteristic colors produced by certain metallic ions when vaporized in a flame.
2. To identify unknown metallic ions by means of their characteristic flame color.
3. To study the similar chemical properties of the elements that belong to the same group on the
periodic table.

Background:
In 1913, Bohr proposed the quantized shell atomic model. His model was a huge success for the
hydrogen atom. He concluded that the electron of the hydrogen atom can only be allowed on certain
energy levels called orbits. Bohr observed four distinct bands in the hydrogen atom’s emission
spectrum. These four bands were related to photons of different colors corresponding to quantized
energy transitions.

The normal electron configuration of atoms or ions of an element is known as the “ground state.” In
this most stable energy state, all electrons are in the lowest energy levels available. When atoms or
ions in the “ground state” absorb energy (heat or other forms of energy), it causes some electrons
“jump” to higher energy levels. The atom is then said to be in the “excited state.” This excited
configuration is unstable, and the electrons “spiral down” to lower energy levels or to its ground state.
During these electronic transitions, electrons release their energies in the form of electromagnetic
radiation. Some of this released energy may be in the form of visible light. The color of this light can
be used as a means of identifying the element. This analysis is known as a flame test. In the case of
the hydrogen atom’s emission spectrum (higher energy level to the lower energy level transitions),
the red color is due to the photons released when excited electrons fall from the 3rd energy level to
the 2nd energy level. All the electronic transitions that fall to the second energy level (Balmer Series)
are visible to humans. Other transitions do exist but are not visible to the human eye.

To perform a flame test of a particular metallic element, its chloride is dissolved in water to prepare an
aqueous solution. The solution is then vaporized in the blue flame of a Bunsen burner. This test
works well for common main group metal ions, and was perfected by Robert Bunsen (1811 – 1899).
Many metallic ions exhibit characteristic colors, e.g. sodium is yellow-orange, and lithium is crimson.

There are three parts in this experiment:

Part A. Emission spectra of the elements using spectral tubes.


Part B. Flame tests.
Part C. Chemical Fingerprinting.
P a g e | 64

Materials:
Set of metal chloride 0.50 M solutions (LiCl, NaCl, KCl, CaCl2, SrCl2, and BaCl2). Bunsen Burner,
wooden splints, striker, cobalt glass plates, well-plates, 0.10 M of (NH4)2CO3, 0.10 M of (NH4)2SO4,
and 0.10 M (NH4)2HPO4 aqueous solutions.

Note: The chloride salts are preferred since they are easily volatilized in the Bunsen burner flame.

Beaker (250 to 500 mL) half-filled with tap water.

Safety:
1. Wear goggles and an apron at all times
2. No eating or drinking is allowed in the chemistry laboratory.
3. Notify your instructor immediately if you touch or accidently ingest any of these solutions.
4. If you experience problems while lighting the Bunsen burner turn then gas line off and ask your
instructor for help.
5. Make sure you wash your hands before you leave the laboratory.
P a g e | 65

PRELAB QUESTIONS NAME: ____________________

1. Identify the initial state and final state of the electron when it absorbs energy.

2. Identify the initial state and final state of the electron when it releases energy.

3. Identify the process by which electrons that surround an atom produce light (or electromagnetic
radiation or energy).

4. Identify the reason chloride salts are used in the Bunsen burner flame.

5. Identify the chloride salt which is shown to have a crimson color when placed in the flame of a
Bunsen burner.
P a g e | 66

Part A. Spectrum Analysis

(Courtesy of rit.edu; creative commons)

Procedure:
1. Obtain the spectral tubes of Hydrogen, Helium, Neon, Oxygen and any others available to you
and place them in the power supply. Then turn on the power and observe the emission spectra
of the samples with a spectroscope and compare the spectra to the chart provided above.

2. Record your observations and compare to the chart above.


___________________________________________________________________________
___________________________________________________________________________
___________________________________________________________________________
___________________________________________________________________________
___________________________________________________________________________
___________________________________________________________________________
___________________________________________________________________________
___________________________________________________________________________
P a g e | 67

Part B. Flame Test


NOTE: Place couple of splints in the unknown solution before all other tests have been completed.

PROCEDURE:

1. Using a striker, light the Bunsen burner and adjust it so that it has a hot blue flame. Partially fill a
250-mL of beaker with tap water.

2. Take a wooden splint, dip it into the LiCl (lithium chloride) solution, and then hold the wooden splint
in the hottest part of the burner flame. Observe the color of the flame. Try not to let the wooden splint
catch on fire. Extinguish the splint in the beaker that contains water.

3. Carefully record your observations in the data table. Be very accurate. Your description of the
color must be accurate enough to distinguish this metal ion from the other ions tested.

4. Repeat with the other solutions. Make sure you follow the order of the metals given in the data
table in Part B, e.g. lithium first and barium last as the sequence follows their location in the periodic
table. Check the color of their flame tests. Record your observations for each solution. When you
examine the sodium and potassium ions, first look at their color alone, then test them again, looking
through a cobalt glass plate.

5. When you have tested all the known solutions and can distinguish the color of each metal ion,
obtain an unknown solution. Determine which metal ion is present by performing a flame test and
comparing this new data to your previous data.

6. Extinguish the splints in a beaker partially filled with water.

NOTE: Place couple of splints in the unknown solution before all other tests have been completed.

Figure 1. The flame colors with different metals present in the solutions .
P a g e | 68
DATE: ____________ NAME: _____________________________

DATA TABLE FOR FLAME TEST.

Metal ion Color of Flame


Lithium
Sodium
Potassium (without
cobalt glass)
Potassium (with
cobalt glass)
Calcium
Strontium
Barium
Unknown # ____

Based on your observations, identify the unknowns you examined:

Unknown # _____ is ________________________________


P a g e | 69

Part C. Chemical Fingerprinting


PROCEDURE:

1. Arrange two well plates according to the figure given below.

2. Using a Beral pipet, drop 5 drops of LiCl in the wells numbered 1, 2, and 3.

3. Place 5 drops of NaCl in the wells numbered 4, 5, and 6.

4. Repeat same procedure for the other solutions.

5. Finally, place 5 drops of the unknown solution in wells 19, 20, and 21.

6. Next, add 5 drops of ammonium carbonate to wells 1, 4, 7, 10, 13, 16, and 19. Then, add 5 drops
of ammonium hydrogen phosphate to wells 2, 5, 8, 11, 14, 17, and 20. Finally, add 5 drops
ammonium sulfate to wells 3, 6, 9, 12, 15, 18, and 21.

7. Record your observations in the grid given on the next page.

NOTE: It may take longer for the CaSO4 to precipitate (step #12).
P a g e | 70
DATE: ________________ NAME: ____________________

DATA TABLE FOR CHEMICAL FINGERPRINTING. *


Note that the unknown is the same sample you used for lame test.

Ammonium Ammonium Ammonium


Solution tested carbonate phosphate sulfate

LiCl

NaCl

KCl

CaCl2

SrCl2

BaCl2

Unknown

*Key: If a precipitate is observed you will record it as ppt and for no reaction, record NR.

Based on the results of your experiments, what metal was found in your unknown? Explain.
Unknown # _____ is ________________________________.
P a g e | 71

POSTLAB QUESTIONS NAME: ____________________

1. Although flame tests are usually specific for each element, they can be misleading. Sodium may be
present in each sample therefore a cobalt glass is needed to observe the real color emitted by
potassium. Why is blue cobalt glass used instead of any other colored glass?

2. An unknown solution tested in this experiment gave a yellow green flame test. Do you expect to
observe a precipitate with the reagents used in Part C of this experiment?

3. Write the electronic configuration of each metal investigated in this module. Does the electron
configuration of each metal predict the reaction outcome observed in chemical fingerprinting?

4. Research some information about the origin of fireworks. Explain how they are made, what
chemicals are used, and what colors they burn.

5. Why do different atoms/ions emit different colors of light?


P a g e | 72
6. Colorful light emissions are applicable to everyday life. Where else have you observed colorful light
emissions?

7. Determine the wavelengths for Potassium (violet, λ = 400 nm) and Strontium (red, λ = 700 nm) light
emissions. Calculate the frequencies (c = wavelength × frequency) and the energy (E = h ×
frequency) for each. Fill the table below with your results. SHOW YOUR WORK!
c = speed of the light = 3×108 m/s and h = Planck’s constant = 6.62×10-34 J×s.

Wavelength (nm) Frequency (Hz) Energy (J)

Potassium

Strontium
P a g e | 74

Experiment 6.
Lewis Dot Structures and VSEPR Models
Objectives:

The objectives of this experiment are:


a) To practice drawing Lewis Structures for various covalently bonded molecules and polyatomic
ions.
b) To use model kits to construct these molecules/ions in order to explore their structure and shapes.
c) To practice predicting molecular shapes (using VSEPR theory).

Background:

Non-metal atoms bond covalently, resulting in the formation of either neutral molecules or polyatomic
ions. A covalent bond is formed when non-metal atoms share their valence electrons, which they do
in order to achieve filled valence orbitals like their nearest noble gas neighbor. This means that most
bonded non-metal atoms will be surrounded by a total of eight valence electrons via the sharing
process – often referred to as the octet rule. A notable exception is hydrogen, which only needs two
electrons to be like its nearest noble gas neighbor, helium. Exceptions to the octet rule will be
discussed later.

A. Lewis Structures:

The Lewis Dot Structures (LDS) of the Main Group Elements are given in the Figure below:

Figure 6.1 The LDS of the main group elements.


P a g e | 75
A Lewis structure (or electron-dot formula) is a two-dimensional structural formula showing the
arrangement of valence electrons around atoms in covalently bonded molecules (or polyatomic ions)
i.e. molecules where nonmetal atoms are held together because they share one or more pairs of
electrons. For example, the Lewis electron dot structure of hydrogen chloride (HCl) is shown below.
nonbonding or lone pair
Bonding pair
.
.. . .. equivalent to
..

Lewis Dot Structure Structural Formula

Note that in the structural formula to the right, the bonding electron pair is represented by a dash
between the two atoms. Each atom is represented by its atom's symbol. Electrons not in a bond
(nonbonding electrons) are generally paired when placed around an atom.

Guidelines for Drawing Lewis Electron-Dot Structures for molecules or polyatomic ions with a
central atom.

These rules can only be used for molecules or polyatomic ions that have a central atom. By a central
atom we mean an atom to which all other atoms in the molecule or polyatomic ion are bonded.

Step 1

Find the total number of valence electrons, i.e. the number of available valence electrons. For a
neutral molecule, the total number of valence electrons is obtained by adding the total number of
valence electrons from all atoms in the molecule. For the main group or representative elements, the
number of valence electrons is equal to the group number (which contains the letter A) in which it is
found on the periodic table e.g. PCl3 has 5 + 3(7) = 26 available valence electrons.
However, for polyatomic ions, you must account for the number of electrons gained or lost, as
indicated by its charge. To obtain the correct number of available valence electrons in a negative
polyatomic ion, you will have to add the magnitude of the charge on the ion, to the total number of
valence electrons from atoms in the ion (e.g. in NO3-1, there are 5 + 3(6) + 1 = 24 available valence
electrons, not 23).
For a positive polyatomic ion, you will have to subtract the magnitude of the charge to the valence
electrons form the atoms in the ions, in order to obtain the correct number of available valence
electrons (e.g. in ammonium ion, NH4+, the available valence electrons is 5 + 4(1) -1 = 8, not 9).

Step 2

Find the central atom and connect the other atoms to it by single bonds. (By a single bond we mean,
sharing a pair of valence electrons).
The central atom can be determined by using the following guidelines.
(a) It is generally the first non-hydrogen atom in the formula.
(b) The atom of lowest electronegativity.
(c) The atom that can form the most single bonds. The number of single bond a main group
nonmetal can form if it obeys the octet rule is equal to
(8 - the group number which contains A).
P a g e | 76
For example, in NO3-1, the central atom is "N". It is the first atom in NO3-1. "N" (EN = 3.0) is the atom of
lower electronegativity as compared with "O" (EO = 3.5). Also, "N" can form the most single bonds
(3) as compared to "O", which can form two single bonds.

Step 3

Find the remaining valence electrons and distribute them in pairs around the various atoms,
completing octets of electrons, but do not start with the central atom (e.g. 6 valence electrons were
used in the three single bonds to each of the oxygen atoms in NO 3-1) so there are 24 - 6 = 18 valence
electrons, left to distribute, as shown below.

.. . ...
. ..
.. . . .. . .. ..
.. ..

Step 4

If there are not enough electrons for each non-hydrogen atom to have an octet, move a nonbonding
pair of electrons from one of the atoms surrounding the central atom (if that atom is capable of
forming more than one single bonds) and make that pair a bonding pair to the atom from which it was
removed i.e. rather than having one shared pair of electrons between the atom from which the
nonbonding pair was moved and the central atom, there are now two shared pairs of electrons or a
double bond. If additional electrons are still required, move another nonbonding pair of electrons and
make it a bonding pair, resulting in a triple bond.

By convention, square brackets are draw around the entire Lewis structure of the polyatomic ion, with
the charge indicated in the upper right hand corner of the square brackets, as shown below.

.. . ..
-1 ..
-1
.. . .. .. . .. . ..
. . .. .
. . .. . . . . . . . ...
.. . . .. .. .. .
.. .
equialent
.. . . . . to .
.. .. ..
P a g e | 77
Note: Three equivalent resonance structures can be drawn for the nitrate polyatomic ion, depending
on which of the oxygen atoms has the double bond.

Step 5

Confirm that the number of valence electrons used in the Lewis dot structure is equal to the number
of available valence electrons.

Exceptions to the octet rule

In some molecules and polyatomic ions, the central atom can have less than an octet of electrons or
more than an octet of electrons. These "exceptions to the octet rule are described below".

a) Beryllium (Be) in BeCl2 and Boron (B) in BCl3 can form molecules in which "Be" has four valence
electrons and "B" has 6 valence electrons. These compounds are generally unstable and very
reactive.

b) Molecules with an odd number of valence electrons, such as NO, are called "free radicals" and are
extremely reactive. N in NO has 7 valence electrons.

c) Some molecules and polyatomic ions have more than eight valence electrons around the central
atom. The central atom can have more than 8 valence electrons, if it is from period 3 or higher
periods, where empty "d" orbitals are available to hold these additional valence electrons. For
example, sulfur which is in period "3" has 12 valence electrons in the compound SF 6.
P a g e | 78
B. Constructing Models

Use your molecular model kit (MolyMod) to construct a three-dimensional model of each of the molecules
indicated in the exercise at the end of this discussion. Color legen is given on the box cover.

Guidelines for constructing Molecules with the model kit.

a. Each colored ball in the kit corresponds to a different atom or group of atoms in the kit. Also, that
ball will have the same number of holes as the number of single bonds that element will form, if it
obeys the octet rule. If that element is in period 3 or a higher period, it is allowed to violate the octet
rule and there will be two types of balls of that color, one with a set of holes that allows it to obey the
octet rule and one with a greater number of holes, allowing that element to form more single bonds
than the octet rule allows.

b. Use a stick or short spring for single bonds.

c. Use two long springs for a double bond and three long springs for a triple bond.

- If the molecule has a central atom, compare each model constructed to the
molecular shape obtained from using valence shell electron pair repulsion (VSEPR)
theory. See Figure 6.2.

- If the molecule does not have a single central atom, compare the arrangement of
atoms around each of so called "smaller molecules with a central atom" that make
up that molecule and see if that is the expected arrangement of atoms, and record
bond angles.

B1. Guidelines for Drawing Models of Molecules and Polyatomic ions.

Sketch a reasonably detailed picture of the model you built.

- Use circles with the symbol of the element in the inside of the circle to represent
atoms.

- Use one solid line to represent a single bond, two solid lines to represent double
bonds, and three solid lines to represent triple bonds.

- Show the relative sizes of angles.

- Use solid wedges, to represent bonds coming out of the plane of the paper, broken
wedges to represent bonds going behind the plane of the paper and solid lines to
represent bonds in the plane of the paper, as shown in the for CH4 and CO, below.
P a g e | 79
B2. Drawings of Models of the molecules CH4 and CO

C. VALENCE SHELL ELECTRON PAIR REPULSION (VSEPR) MODEL

Lewis structures show the two-dimensional distribution of atoms and electrons. The molecular
geometry, or three-dimensional shape of a molecule or polyatomic ion, can be determined by valence
shell electron pair repulsion (VSEPR) theory. By the molecular geometry or shape of a molecule, we
mean the arrangement of atoms in space around a central atom. Thus, we can talk about the overall
shape of a molecule that has a central atom.

The basic principle in VSEPR theory is that valence electron pairs arrange themselves around a
central atom so as to stay as far apart as possible, to minimize repulsion.

The molecules in which the central atom does not possess lone pair(s) have the same electron
distribution and molecular shape.

C1. General rules for the VSEPR model

- Each area where electrons exist around the central atom is called an electron
domain or simply domain. Single, double, and triple bonds each occupy one electron
domain.

- A nonbonding electron pair is considered, like a single bond, double bond or triple
bond, to occupy one electron domain.

- In VSEPR theory, letters are used to represent general formulas of compounds.

These are:

A - This is the central atom of the molecule.


X - This letter represents the atoms attached to the central atom. No distinction is made between
atoms of different elements. For example, AX4 can refer to CH4 or CBr4.
E - It stands for nonbonding electron pairs (lone pairs).
P a g e | 80
C2. Guidelines for applying the VSEPR Model

1. Draw the Lewis electron-dot formula.

2. Count the number of electron domains (number of outer atoms, X + number of lone pairs, E)
around the central atom, A.

3. Determine the number of lone pair(s), E.

4. Determine the general formula, and match it up with the molecular geometry obtained in Figure
6.2.

For example, consider the compound, NH3, whose Lewis electron dot formula is shown below:

.
.. ...
NH3 H2O

This molecule has a general molecular formula AX3E. NH3 has 4 electron domains, 3 electron
domains due to the 3 atoms of Hydrogen (X3) plus the one electron domain due to the one lone pair
(E) on the central Nitrogen atom, N (A). The arrangement of the 4 electron domains will tetrahedral,
but the molecular shape or molecular geometry of AX3E, is "trigonal pyramidal".

Likewise, consider the Lewis dot structure of compound, H2O, as shown above. The molecule has a
general molecular formula, AX2E2. It has 4 electron domains, 2 electron domains due to the 2 atoms
of Hydrogen (X2) plus the two-electron domains due to the two lone pairs (E2) on the central oxygen
atom, (A). The arrangement of the 4 electron domains will be tetrahedral, but the molecular shape or
molecular geometry of AX2E2, is "Bent" or "angular".
P a g e | 81

Figure 6.2. Electron pair and molecular geometry. Note that a molecule made
by two atoms, AX, is linear.
P a g e | 82

PRELAB QUESTIONS NAME: ________________________

General
Total Sketch of Molecular
Number Lewis Dot Structure Shape of Formula Molecular
Formula of Molecule Shape
Valence AXE
electrons Designation

CH4

AlCl3

N2

H2O
P a g e | 83

WORKSHEET NAME: __________________

Total LEWIS ELECTRON Drawing of General Molecular


Formula Number DOT STRUCTURE Molecular Molecular Geometry
of valence model Formula or
electrons Molecular
shape

o
NH3 8 o Trigonal
o o AX3E pyramidal
..

H2

Cl2

O2

CO2

CBr4
P a g e | 84

Total LEWIS ELECTRON Drawing of General Molecular


Formula Number DOT STRUCTURE Molecular Molecular Geometry
of valence model Formula or
electrons Molecular
shape

PF3

HCN

NO3-

CO3-2

H3O+1

OH-1
P a g e | 85
Total Molecular
Number LEWIS ELECTRON Drawing of General Geometry
Formula of valence DOT STRUCTURE Molecular Molecular or
electrons model Formula Molecular
shape

SF2

Does not follow the


octet rule

SF4

Does not follow the


octet rule

SF6
P a g e | 87

Experiment 7.
Qualitative Analysis of Anions
LEARNING OBJECTIVES
The students will be able
1. to observe the chemical behavior of various anions in solutions.
2. to apply various laboratory techniques.

BACKGROUND
This experiment is an exercise in qualitative analysis where you will be analyzing solutions to
determine the presence of anions. Certain techniques will be used to separate the ions from one
another. A good starting point for this type of analysis is the observation of the color and solubility of a
substance. Therefore, we will rely on more sophisticated analyses such as precipitation, acid-base,
and electron transfer reactions. Again, the goal will be to correctly identify the anions (two of the five
investigated) present in an unknown solution.

 In the first part, you will analyze a “known” solution which contains all five of the anions; these
procedures will enable you to see the color changes, gas evolution, precipitation, and any
other special effects of each reaction.
 In the second part, you will analyze an “unknown” solution using the same techniques, to
determine the presence of two of these anions.
 The plan should be simple.
First, you will investigate the precipitation reactions given in Table 1. Remarkably, the AgNO 3
solution will give precipitates with all of the anions present in the solution. The only anion that
gives gas evolution is carbonate. At some point during the analysis each anion will react
specifically with the reagent added to the sample. Therefore, you will develop a schematic
flowchart called “Flowcharting Qualitative Analysis”.
P a g e | 88

MATERIALS
Known and unknown solutions: 0.10 M NaCl, 0.10 M KI, 0.50 M Na2CO3, 0.10 M Na2SO4, and 0.10 M
NaBr.

Test Reagent:

Silver nitrate, 0.10 M AgNO3, nitric acid, 6.0 M HNO3, Ammonia, 6.0 M NH3, iron(III) nitrate, 0.10 M
Fe(NO3)3 in 6.0 M HNO3, Potassium permanganate, 0.10 M KMnO4, Barium chloride, 0.10 M BaCl2,
mineral oil, baking soda (to neutralize spills)

Equipment: small test tubes (12), test tube holder, test tube rack, Beral pipets, centrifuge.

SAFETY ALERT:

 Most of the acids and bases are very concentrated and can cause chemical burns.
 Acid spills can be neutralized with baking soda, NaHCO3.
 Neutralize base spills with vinegar solution (dilute acetic acid, HC2H3O2)
 Some compounds are poisonous; wash your hands.
 Solutions containing silver and potassium permanganate can cause stains which do not
appear immediately.

NOTE: DISPOSE OF THE MINERAL OIL LAYER IN THE ORGANIC WASTE CONTAINER.
P a g e | 89

Reagent AgNO3 Ba(NO3)2 HNO3 Fe(NO3)3 and KMnO4 and Comments


Anion mineral oil mineral oil
White NR NR The
Chloride, Cl- precipitate, precipitate is
AgCl soluble in
conc. NH3
NR NR Mineral oil The
Bromide, Br- Beige will turn precipitate is
precipitate, orange. slightly
AgBr soluble in
conc. NH3
White The
Carbonate, precipitate, Precipitate CO2 gas precipitate is
CO32- Ag2CO3 NOT soluble
in conc. NH3
Yellow NR NR Mineral oil The
Iodide, I- precipitate, will turn precipitate is
AgI purple NOT soluble
in conc. NH3
White Precipitate NR The
Sulfate, SO42- precipitate, precipitate is
Ag2SO4 NOT soluble
in conc. NH3

Table 1. Summary of the chemical reactions.


P a g e | 90

PRELAB QUESTIONS NAME: _______________________

1. Identify the five possible anions which are to be identified in this experiment.

2. How many of these five anions will appear in your unknown sample?

3. Four of the chemicals used to make the known and unknown samples contain the sodium ion.
How will this affect your results in this experiment?

4. What purpose does the centrifuge have in this experiment?

5. Four of positive tests for an unknown ion involve the formation of a precipitate. Which ion has a
positive test that involves the formation of gas?
P a g e | 91

PART A. KNOWN SOLUTION OF THE ANIONS


PART I

1. Aqueous solutions of all of the anions to be tested are colorless. The cation associated with
each of the anions will be either sodium or potassium ion.

Using a Beral Pipet, place 40 drops of the known sample (colorless solution) into a small test
tube.

2. Add 2-3 drops of 6 M HNO3 solution. Evolution of CO2 gas indicates the presence of carbonate
ion. The gas evolution could be difficult to observe. Record your observations. The
corresponding net ionic equation representing the chemical reaction is:
CO32-(aq) + 2 H+(aq)  CO2(g) + H2O(l)

Figure 1. CO2 evolution when carbonate ions are present.

3. Add 10 drops of 0.20 M of Ba(NO3)2 solution to the same test tube. A white precipitate will
form. This is a positive identification for the sulfate ion. Observe the reaction. The
corresponding net ionic equation representing the chemical reaction is:

Ba2+(aq) + SO42-(aq)  BaSO4(s)

Figure 2. Precipitation of Barium cations with sulfate ions.

4. Place the test tube in the centrifuge and counterbalance it with a test tube of the same size
filled with water to the same level (this is very important otherwise the centrifuge can move and
cause undesired consequences). Centrifuge for 5-10 minutes at 2000 rpm. DO NOT OPEN
THE COVER OF THE CENTRIFUGE UNTIL THE CENTRIFUGE COMES TO A FULL STOP.
Remove the test tube from the centrifuge. You will see a solid at the bottom of the test tube.
This a positive identification of the presence of the sulfate ion.
P a g e | 92
5. Add the supernatant solution to a different test tube. Add 20 drops of 0.10 M of AgNO3
solution. Centrifuge the test tube for 5-10 minutes at 2000 rpm. (Counterbalance the test tube
using another test tube of the same size filled with water to the same level; this is very
important otherwise the centrifuge can move and cause undesired consequences). You will
observe white yellow precipitate at the bottom of the test tube. The chloride, bromide, and
iodide ions are all precipitated. Observe the reaction and write the corresponding equations
representing the chemical reactions given below (see the note to the students).

Figure 3. Swinging basket centrifuge (left) and colors of the halide precipitates.
(Courtesy of edusanjalbiochemist.com)

6. Add 6.0 M NH3 solution to dissolve AgCl. AgBr and AgI will stay in the solid. Carefully remove
the liquid form the test tube and place into another test tube. Add HNO 3, test with litmus or pH
paper to see if the solution is acidic. If it is not, add more HNO 3 until the solution is acidic. This
will precipitate AgCl and positively confirm the presence of the chloride ions.

PART II

7. In a clean test tube add 20 drops of the known solution. Add 10 drops of 0.10 M Fe(NO3)3
solution. Swirl the mixture. Add 10 drops of mineral oil and shake the mixture. The mineral oil
(the upper layer) will turn purple indicating the presence of the iodide ions.

8. In a clean test tube add 20 drops of the known solution. Add 2 drops 6 M HNO3 solution and 10
drops of 0.10 M KMnO4 solution. Swirl the mixture. Add 10 drops of mineral oil and shake the
mixture. The mineral oil (the upper layer) will turn yellow-orange indicating the presence of the
bromide ions.
P a g e | 93
Note to students: The halides all form insoluble silver compounds. Silver chloride is a white
solid, silver Bromide is pale cream−colored solid, and the solid silver iodide is light yellow in
color. The net ionic reactions are given below:
.
Ag+ + Cl-  AgCl(s) Ag+ + Br-  AgBr(s) Ag+ + I-  AgI(s)

Silver chloride is the only silver halide that dissolves in 6.0 M ammonia, NH3, forming the
colorless, [Ag(NH3)2]+ complex ion. If nitric acid, HNO3, is added to a solution containing this
ion, the ammonia in the complex reacts with hydrogen ions to form ammonium ions, and the
silver recombines with the chloride ions that are still in solution.

AgCl(s) + 2NH3(aq) → [Ag(NH3)2]+(aq) + Cl-(aq)

[Ag(NH3)2]+(aq) + Cl-(aq) → AgCl(s) + 2NH4 +(aq)

In acid solution, iron(III) ion, Fe3+, is a weak oxidizing agent capable of oxidizing the
easily oxidized iodide ion to iodine. Bromide and other ions present will not interfere.
The nonpolar iodine will preferentially dissolve in nonpolar mineral oil, where it can be
identified by its pink to violet color.

2I-(aq) + 2Fe3+(aq) → I2(aq) + 2Fe2+(aq)

KMnO4 is a stronger oxidizing agent than the iron(III) nitrate and will oxidize bromide, Br-
to bromine, Br2. Other ions present will not interfere. The nonpolar bromine can be
extracted into nonpolar mineral oil where it can be identified by its characteristic yellow
to brown color.

10Br-(aq) + 2MnO4 -(aq) + 16H+(aq) → 5Br2(aq) + 2Mn2+(aq) + 8H2O(l)


P a g e | 94

PART B. FLOWCHART FOR THE UNKNOWN ANALYSIS


UNKNOWN # _____________

DATE: ____________ NAME: _____________________________

Figure 4. Flowchart of the anion analysis.

NOTE:
NOT

REPORT: The unknown sample # ________ contains ____________ and ___________ ions.
P a g e | 95

POSTLAB QUESTION NAME: ___________________

1. Write the name of the following ions.

a. Cl-

b. Br-

c. I-

d. SO42-

e. NO3-

f. C2H3O2-

g. CO32-

h. NH4+

i. Ba2+

j. Cu2+

k. Fe3+

2. Write the formula of the following ions.

a. hypochlorite

b. chlorate

c. magnesium

d. sulfite

e. bicarbonate

f. aluminum

g. sulfide

h. fluoride

i. nitride

j. phosphate

k. cobaltic or cobalt(III)
P a g e | 96
3. Write the formulas of the following compounds.

a. magnesium acetate

b. sodium sulfate

c. iron(III) sulfite

d. potassium bicarbonate

e. zinc oxide

f. copper(II) oxide

g. sulfur dioxide

h. dinitrogen monoxide

i. aluminum phosphate

j. magnesium nitride

k. sodium oxalate

4. Write the name of the following formulas.

a. Mg3P2

b. Ca(HCO3)2

c. CaF2

d. Al2(CO3)3

e. K3PO3

f. FeCl3

g. Ni2S3

h. CaC2O4

i. Zn(C2H3O2)2

j. CrO3

k. MnO2
P a g e | 97
5. A student analyzed an unknown sample that contained a single anion. The sample gave a yellow
precipitate upon addition of a solution of AgNO3 which did not dissolve in ammonia. Which anion is
present in the unknown solution? Write the corresponding net ionic equation.

6. A student analyzed an unknown sample that contained a single anion. The sample gave a white
precipitate upon addition of a solution of AgNO3 which dissolved in ammonia. Which anion is present
in the unknown solution? Write the corresponding net ionic equation.

7. Iodine (I2) and bromine (Br2) are soluble in mineral oil and slightly soluble in water. Why? Suggest
another solvent, hexane or alcohol, which can be used instead of mineral oil? Explain.

CH3CH2CH2CH2CH2CH3 CH3CH2OH
Hexane Alcohol
P a g e | 98
8. Write the net ionic equations for the following reactions.

a. An aqueous solution of sodium carbonate is added to an aqueous solution of calcium chloride.


Calcium carbonate is precipitated.

b. An aqueous solution of sodium phosphate is added to an aqueous solution of strontium chloride.


Strontium phosphate is precipitated.

c. An aqueous solution of potassium sulfate is added to an aqueous solution of barium chloride.


Barium sulfate is precipitated.

d. An aqueous solution of ammonium iodide is added to an aqueous solution of silver nitrate. Silver
iodide is precipitated.
P a g e | 100

Experiment 8.
Water of Hydration
You may have noticed that salt shakers in restaurants contain rice grains. Did you ever wonder why?
The rice absorbs moisture faster and more efficiently than the salt does. It prevents the salt from
caking up in the shaker due to high humidity. Most solid chemical compounds will absorb some water
if they have been exposed to the atmosphere for any length of time. In most cases, the water is
present in very small amounts and is absorbed on the surface of the crystals. Some ionic compounds
contain large amounts of water that is chemically bound in the crystal. The water that is present in
these salts is called water of hydration and is usually bound to the cations in the salt.

The water molecules in a hydrate are removed relatively easily. In most cases simply heating a
hydrate to a temperature somewhat above the boiling point of water will drive off the water of
hydration. Hydrated copper(II) sulfate is typical in this regard; it is converted to anhydrous salt. This
reversible process is endothermic (requires heat) and is followed by the color change from blue to
gray-white. The process can easily be reversed by adding water.

The chemical reaction is represented by the following equation:

CuSO4.5H2O(s)  CuSO4(s) + 5H2O(l)


Blue White

In the dehydration reaction, the crystal structure of the solid will change. On heating the hydrated
CuSO4.5H2O crystals are converted to a grayish white powder, the anhydrous salt.

Some hydrates lose water to the atmosphere upon standing. This process is called efflorescence.
The amount of water lost depends upon the amount of water in the air, as measured by its relative
humidity. In moist, warm air, CoCl2 is fully hydrated and is red; in dry cold air CoCl2 loses most of its
water of hydration and is blue; at intermediate humidities CoCl2 exists as a dihydrate and is violet.

Some anhydrous ionic compounds will tend to absorb water from the air or other sources so strongly
that they can be used to dry liquids or gases. These substances are called desiccants, and are said
to by hygroscopic. A few ionic compounds can take up so much water from the air that they dissolve
in the water they absorb; sodium hydroxide, NaOH will do this. This process is called deliquescence.
Eventually, if it is left exposed to air, NaOH will dissolve itself in the water present in the atmosphere.
All ionic hydrates are readily soluble in water. The amount of bond water may depend upon the way
the hydrate is prepared, but in general the number of moles of water per mole of ionic compound is
an integer. Dehydration is a different process than efflorescence. In efflorescence, salt dissolved in
water migrates to the surface of a porous material. The water evaporates and leaves a coating of the
salt.

There are two parts in this experiment:

In Part A, you will verify the percent of water of a known hydrate such as CuSO4.5H2O. In Part B, you
will be asked to determine the amount of water lost by a sample of unknown hydrate upon heating.
From this amount, if given the formula or the molar mass of the anhydrous sample, you will be able to
calculate the formula of the hydrate itself.
P a g e | 101

SAFETY INSTRUCTIONS

1. All
the chemical used in this experiments are harmful. You MUST wash your hands after you finish
the experiment.
2. Wear your eye goggles at all times.
3. During heating the hydrate cover the beaker with the watch glass.
4. Do not touch the heater during the experiment. When the experiment is finished ask your instructor
to remove the heater from the workbenches.
5. Do NOT eat or drink in the laboratory.

MATERIALS

Hot plates, stand, O-ring, beakers (150-mL), watch glass, copper(II) sulfate pentahydrate, barium
chloride dehydrate, zinc sulfate heptahydrate, magnesium sulfate heptahydrate.

Barium chloride, along with other water-soluble barium salts, is highly toxic. Zinc sulfate
powder and cupric sulfate are eye irritants. Magnesium sulfate is commonly known to be safe.
Magnesium sulfate is a common mineral pharmaceutical preparation of magnesium, commonly
known as Epsom salt, used both externally and internally. Epsom salt is used as bath salts and for
isolation tanks. Oral magnesium sulfate is commonly used as a saline laxative or osmotic purgative.
P a g e | 102

PRE-LABORATORY QUESTION: Water of Hydration

A worked example for the lab report.

A student is given a sample of a green nickel sulfate hydrate. The mass of the empty beaker and
watch glass are found to be 21.244 g. After heating the beaker to drive off the water of hydration, she
then lets the beaker cool, and finds it has a lower mass; the beaker, cover and contents now weigh
21.854 g. After the second heating, the student finds that the mass is 21.841 g. In the process the
sample was converted to yellow anhydrous NiSO4.

1. Mass of the empty beaker and the watch glass: 21.244 g

2. Mass of the beaker, watch glass, and hydrate: 22.326 g

3. Mass of the hydrate: 1.082 g


(line 2) – (line 1)

4. Mass of the anhydrous salt, beaker, and the 21.854 g


watch glass after the first heating:

5. Mass of the anhydrous salt, beaker, and the 21.841 g


watch glass after the second heating:

6. Mass of the water: 0.485 g


(line 2) – (line 5)

8. Percent of water in the hydrate: 44.82 %


[(line 6) / (line 3)] × 100
P a g e | 103

PRELAB QUESTIONS NAME: _________________

1. The final products in this experiment will weigh more or less than the starting material.

2. Identify what has been lost from the starting material via the heating process.

3. Is the process performed in this experiment exothermic or endothermic?

4. Does the anhydrous salt contain any water?

5. Is the final product a hydrate or anhydrous?


P a g e | 104
6. Calculate the percent of water in

a. BaCl2.2H2O Ba = 137; Cl = 35.5; O = 16; H = 1

b. ZnSO4.7H2O Zn = 65; S = 32; O = 16; H = 1

c. MgSO4.7H2O Mg = 24.3; S = 32; O = 16; H = 1

7. A sample of 5.00 g of FeCl3.xH2O is heated, driving off 2.00 g of water. What is the empirical
formula of the hydrate?

8. A sample of 8.85 g of Na2CO3 hydrate is heated, losing 1.28 g of water. Find the empirical formula.
P a g e | 105

EXPERIMENTAL PROCEDURES

PART A. Percent water in copper(II) sulfate pentahydrate CuSO4.5H2O

1. Weigh the empty beaker and the watch glass and record the mass.

2. Place approximately 2.00 grams of the blue sample into the beaker, cover with the watch glass,
and weigh it again. Record the mass.

3. Uniformly disperse the hydrate and make sure there are no lumps present. Adjust the temperature
of the heater to 300 (right knob) and start heating the beaker and its contents.

4. Carefully wipe the water condensed to prevent it from dripping back into the beaker.

5. When condensation is no longer visible, stop heating. Wait until the beaker is cooled. If you need
help ask your instructor for assistance. Carefully remove the beaker from the heater and weigh the
beaker. Record the mass.

6. Place the beaker and the watch glass on the heater and start heating again. After 10 minutes, stop
heating. Wait until the beaker and the watch glass are cooled, then remove them from the heater and
weigh them. Record the mass.

7. Repeat step (6) and record the mass.

8. You need to repeat Step (6) until you have a constant mass. This is necessary to make sure that
all the water contained in the hydrate is released.
P a g e | 106

PART B. Percent water in the unknown hydrate

You will repeat the experiment using an unknown hydrate. After determining the percent water in the
hydrate, you will be able to identify the unknown. Now, obtain an unknown from your instructor and
record the number of the unknown. The possible hydrates are BaCl2.2H2O, ZnSO4.7H2O, and
MgSO4.7H2O.

1. Weigh the empty beaker and the watch glass and record the mass.

2. Place about 2.00 grams of the unknown sample into the beaker, cover with the watch glass, and
weigh it again. Record the mass.

3. Uniformly disperse the hydrate and make sure there are no lumps present. Adjust the temperature
of the heater to 250 (right knob) and start heating the beaker and its contents.

4. Carefully wipe the water condensed to prevent it from dripping back into the beaker.

5. When condensation is no longer visible, stop heating. Wait until the beaker is cooled. If you need
help ask your instructor for assistance. Carefully remove the beaker from the heater and weigh the
beaker. Record the mass.

6. Place the beaker and the watch glass on the heater and start heating again. After 10 minutes stop
heating. Wait until the beaker and the watch glass are cooled, then remove them from the heater and
weigh them. Record the mass.

7. Repeat step (6) and record the mass.

8. You need to repeat Step (6) until you have a constant mass. This is necessary to make sure that
all the water contained in the hydrate is released.
P a g e | 107
DATE: ________________ NAME: ___________________

DATA SHEET FOR PART A.


TRIAL 1 TRIAL 2

1. Mass of the empty beaker and the watch glass: __________ g __________ g

2. Mass of the beaker, watch glass, and hydrate: __________ g __________ g

3. Mass of hydrate: __________ g __________ g


(line 2) – (line 1)

4. Mass of the anhydrous salt, beaker, and the __________ g __________ g


watch glass after the first heating:

5. Mass of the anhydrous salt, beaker, and the __________ g __________ g


watch glass after the second heating:

6. Mass of the water: __________ g __________ g


(line 2) – (line 5)

7. Percent of water in the hydrate: __________ % __________ g


[(line 6) / (line 3)] × 100

8. Average percent of water in the hydrate: _____________ %

9. Compare (line 8) to the correct value of 36.1%.

Your value  36.1


Percent Error = | |  100 =
36.1

Note: If the percent water is too low, you can heat one more time.
P a g e | 108
DATE: ________________ NAME: ___________________

DATA SHEET FOR PART B.


UNKNOWN SAMPLE _______________

TRIAL 1 TRIAL 2

1. Mass of the empty beaker and the watch glass: __________ g __________ g

2. Mass of the beaker, watch glass, and hydrate: __________ g __________ g

3. Mass of hydrate: __________ g __________ g


(line 2) – (line 1)

4. Mass of the anhydrous salt, beaker, and the __________ g __________ g


watch glass after the first heating:

5. Mass of the anhydrous salt, beaker, and the __________ g __________ g


watch glass after the second heating:

6. Mass of the water: __________ g __________ g


(line 2) – (line 5)

7. Percent of water in the hydrate: __________ % __________ %


[(line 6) / (line 3)] × 100

8. Average percent of water in the hydrate: _____________ %

9. Identification of the unknown hydrate: ________________________


(Refer to prelab question #6)

Note: Possible hydrates are BaCl2.2H2O, ZnSO4.7H2O, and MgSO4.7H2O.


P a g e | 109

POSTLAB QUESTIONS NAME: ___________________


1. What is the percent water in calcium chloride dihydrate, CaCl2.2H2O?

2. A 2.50-gram sample of CuSO4.xH2O is heated, releasing the water. After heating, 1.59 g of the
anhydrous salt (CuSO4) remains. What is the empirical formula of the hydrate?

3. A hydrate is found to have 48.8 g MgSO4 and 51.2 g H2O. What is the empirical formula?
P a g e | 110
4. A 13.52 g sample of Magnesium Sulfate hydrate is heated in a crucible. The crucible contains 6.60
g of anhydrous salt after the second heating. What is the formula of the hydrate? Mg = 24.3; S = 32;
O = 16; H = 1.

5. A 5.00 g sample of NiCl2.xH2O is heated, driving off 2.28 g of water. What is the empirical formula
of the hydrate? Ni = 58; Cl = 35.5; H = 1; O = 16.

6. A 1.74 g sample of K2CO3 hydrate is heated, losing 0.36 g of water. Find the empirical formula of
the hydrate. K = 39; C = 12; O = 16; H = 1.
P a g e | 111
7. In Part A of the experiment, you reported a % value which is lower than 36.1%. Briefly discuss
what may have gone wrong?

8. In Part A of the experiment, you reported a % value which is higher than 36.1%. Briefly discuss
what may have gone wrong?
P a g e | 113

Experiment 9.
Types of Chemical Reactions
LEARNING OBJECTIVES:
The students will be able:
1. to make observations and classify the reaction.
2. to identify the type of reaction and predict the product(s).

BACKGROUND:

A chemical reaction is a process that is usually characterized by a chemical change in which the
starting materials (reactants) are different from the products. Chemical reactions tend to involve the
motion of electrons leading to the breaking the bonds and forming of new bonds. There are several
different types of chemical reactions and more than one way of classifying them.
Several observations can be made during chemical reactions:

a. heat evolution
b. light
c. change in smell
d. change in color
e. gas evolution
f. precipitation

Often, a combination of the above observations can be observed in synergy, e.g. the reaction can
give off gas and also release heat. A reaction that release heat is called exothermic and the one that
absorbs heat is called endothermic. In general, a reaction that releases heat and gives off gas
happens at room temperature.

These are the four most common types of chemical reactions:

A. Synthesis or combination
B. Decomposition
C. Single replacement
D. Double replacement or metathesis

In addition, there are two more types of reactions known as neutralization (double replacement
reactions between acids and bases) and oxidation-reduction reactions. Combustion where carbon is
oxidized by oxygen to produce carbon dioxide is a typical combination reaction.
P a g e | 114
Types of Involve
chemical General Examples Example reactions Comments Electron
Transfer
reactions using symbols
Elements or
compounds
combine to
Synthesis A + B  AB 2Mg + O2  2MgO form a larger Sometimes
compound

A complex
compound
breaks apart
Decomposition AB  A + B 2H2O2  2H2 + O2 to elements Sometimes
or simpler
compounds

A single
Single AB + C  A + CB CuCl2 + Zn  Cu + ZnCl2 element
replacement replaces the Always
less active
one in the
compound
Ions in
Double AB + CD  AD + CB AgNO3 + NaCl  AgCl + NaNO3 reacting No
replacement compounds
switch places

Redox 2Na + Cl2  2NaCl Always

Combustion C + O2  CO2 Always

Table 1. Summary of the chemical types of reactions and specific example


P a g e | 115
SAFETY INFORMATION:

1. WEAR THE EYE GOGGLES AT ALL TIMES.


2. DISPOSE OF THE CHEMICAL ACCORDING TO THE DIRECTIONS OF YOUR INSTRUCTOR.
3. WASH YOUR HANDS AFTER FINISHING THE EXPERIMENT.
4. NO EATING or DRINKING IN THE LABORATORY.

MATERIALS AND EQUIPMENT:

Well plates, test tubes, test tube rack, test tube holder (clamp), crucible tongs, stirring rod spatula,
waste beaker, dropped bottles with various chemicals, Bunsen burner and flint striker, watch glass,
wooden splints, safety glasses.

CHEMICALS:

Magnesium ribbon, copper metal, zinc metal, 0.10 M zinc sulfate, ZnSO4, 0.10 M iron(II) sulfate,
FeSO4, 0.10 M magnesium sulfate, MgSO4, 0.10 M sulfuric acid, H2SO4, 0.10 M hydrochloric acid,
HCl, 0.10 M sodium hydroxide, NaOH, phenolphthalein indicator, hydrogen peroxide, manganese
dioxide, 0.10 M potassium iodide, KI, 0.05 M silver nitrate, AgNO3, sodium carbonate, Na2CO3,
ethanol, C2H5OH.
P a g e | 116

PRELAB QUESTIONS NAME: _________________

1. Identify the six types of reactions that will be investigated in this experiment.

2. Identify four ways to identify if a chemical reaction has occurred.

3. Some reactions require a Bunsen burner for the reaction to occur. Would you classify these as
exothermic or endothermic reactions?

4. What type of reaction occurs when a zinc rod is immersed in an aqueous copper(II) sulfate
solution?
P a g e | 117

PART A: Combination (synthesis) reactions: A + B  AB


PROCEDURE:

1. Obtain a small piece of Magnesium ribbon.

2. Set-up and light a Bunsen burner in a fume hood.

3. Hold onto one end of the magnesium ribbon with a pair of tongs.

4. Ignite the magnesium by placing the end of the magnesium ribbon into the flame.

5. Record your observations.

CAUTION: When the magnesium ignites it will release ultraviolet light which will harm your
eyes. Do not stare directly into the light.

Reactants Products Balanced Equation Observation


P a g e | 118

PART B. SINGLE REPLACEMENT: METAL REACTIVITIES.


PROCEDURE:

1. Obtain a 4 × 6 well plate.

2. Add a few drops of the solutions to the vertical wells as described on the figure above.

3. Place a small piece of the different metals in each of the solutions.

4. After 5-10 minutes, record your observations.


P a g e | 119

DATA TABLE FOR PART B. NAME: ____________________

Place the letter R in the table below if you observe a reaction and NR if no Reaction is observed.

CuSO4 FeSO4 ZnSO4 MgSO4

Cu

Fe

Zn

Mg

For each reaction that occurred, write a balanced equation and an observation statement that
describes each product of the reaction in each well.

CuSO4 FeSO4 ZnSO4 MgSO4

Cu

Fe

Zn

Mg
P a g e | 120

PART C: DECOMPOSITION REACTION. AB  A + B

THIS REACTION MUST BE PERFORMED IN THE FUME HOOD!

Decomposition reactions are characterized by one substance breaking down to two or more simpler
substances.

Reaction: Decomposition of hydrogen peroxide (H2O2)

1. Add small amount (using the tip of the spatula) of manganese dioxide, MnO2, in a large test tube.
Remember, a catalyst is not used up in the reaction and its formula is written above the arrow.

2. Add one drop of 0.1 M HCl. Shake the test tube.

3. Hold the test tube with a test tube holder. Add 1-2 mL of hydrogen peroxide to the test tube.

4. Place your thumb over the mouth of the test tube to trap the gas. Wait a few seconds.

5. Test the gas that is being given off by placing a glowing splint into the test tube. (Your partner will
light a wooden splint and extinguish the flame. Place the glowing splint into the test tube. the wooden
splint will ignite, indicating the presence of oxygen as a product of the decomposition reaction)

7. Write a complete balanced equation for this reaction.

Reactant(s) Products Balanced Equation Observation


P a g e | 121

PART D. DOUBLE REPLACEMENT REACTIONS: AB + CD  AD + CB


In double replacement reactions, the substances dissolve in water and dissociate to form ions. The
ions are free to move. The ions in the reacting substances will switch places and a precipitate, a solid
compound which is insoluble in water, will form. If a gas is formed, you will see bubbles.

Reaction 1: Ammonium hydrogen phosphate reacts with magnesium chloride.

1. Add 10 drops of 0.10 M magnesium chloride to a test tube.

2. Add 10 drops of 0.10 M Ammonium hydrogen phosphate to the test tube.

3. Record your observations.

4. Write a complete balanced equation for this reaction.

Check the solubility table to identify the insoluble product. Draw an arrow pointing down or use the (s)
To indicate the precipitate in this reaction.

Reactant(s) Products Balanced Equation Observations

Reaction 2: Silver nitrate reacts with potassium iodide

1. Add 10 drops of 0.10 M silver nitrate to a spot plate.

2. Add 10 drops of 0.10 M potassium iodide to the spot plate.

3. Record your observations.

4. Write a complete balanced equation for this reaction.


Note: Place the yellow solid in a designated waste container as instructed by your instructor.

Reactant(s) Products Balanced Equation Observations


P a g e | 122
Reaction 3: Sodium hydrogen carbonate (NaHCO3) reacts with hydrochloric acid

1. Place about 0.5 gram (tip of spatula) of sodium hydrogen carbonate into a test tube.

2. Add 10 drops of 0.10 M HCl to the test tube.

3. Test the gas that is being given off by placing a glowing splint into the tube.

4. Record your observations.

5. Write a complete balanced equation for this reaction.

Reactant(s) Products Balanced Equation Observation


P a g e | 123

PART E. NEUTRALIZATION REACTION.


1. In a small test tube, place 10 drops of 0.10 M of HCl.

2. Add 1 drop of Phenolphthalein indicator to the test tube.

3. Observe the color.

4. Using a dropper, add 0.10 M of NaOH solution drop by drop until you observe a color change.
Count and record the number of drops required.

5. Record your observations.

Reactant(s) Products Balanced Equation Observations

PART F: COMPLETE COMBUSTION. INSTRUCTOR DEMO!


C2H5OH + O2  CO2 + H2O

Reaction: The combustion of ethanol

1. Place 5-10 drops of ethanol (C2H5OH or CH3CH2OH) onto a watch glass.

2. Light a wooden splint and place it near the ethanol.

3. Record your observations.

4. Write a complete balanced equation for this reaction.

Reactant(s) Products Balanced Equation Observation

C2H5OH + O2 CO2 + H2O


P a g e | 124

POSTLAB QUESTIONS NAME: _____________________

1. Identify the type of reaction shown in each of the following chemical equations:

a. 2NaClO3(s)  2NaCl(s) + 3O2(g) _____

b. AgNO3(aq) + LiCl(aq)  AgCl(s) + LiNO3(aq) _____

c. N2(g) + 3 H2(g)  2 NH3(g) _____

d. 2AgNO3(aq) + K2SO4(aq)  Ag2SO4(s) + 2KNO3(aq) _____

e. NH4HCO3(s)  NH3(g) + CO2(g) + H2O(g) _____

f. H2SO4(aq) + 2NaOH(aq)  Na2SO4(aq) + 2H2O(l) _____

g. CaO(s) + CO2(g)  CaCO3(s) _____

h. 2AgNO3(aq) + Cu(s)  Cu(NO3)2(aq) + 2Ag(s) _____

i. CaCO3(s)  CaO(s) + CO2(g) _____

j. 2NaI(aq) + Cl2(g)  2NaCl(aq) + I2(s) _____

k. 2Ca(s) + O2(g)  2CaO(s) _____


P a g e | 125
2. For the reaction between zinc metal and hydrochloric acid (aqueous) producing zinc chloride
(aqueous) and hydrogen gas

(a) write an equation or this reaction

(b) balance the equation from Part (a).

(c) classify the reaction.

3. For the reaction between aqueous barium nitrate and aqueous sodium sulfate producing the
precipitate (solid) barium sulfate and aqueous sodium nitrate

(a) Write an equation for this reaction.

(b) Balance the equation from Part (a).

(c) Classify the reaction.


P a g e | 126
4. Nitrogen molecules and fluorine molecules react to form nitrogen trifluoride gas.

(a) Write an equation for this reaction.

(b) Balance the equation from Part (a).

(c) Classify the reaction.

5. Solid sodium hydrogen carbonate is gently heated producing carbon dioxide gas, water vapor, and
sodium carbonate.

(a) Write an equation for this reaction.

(b) Balance the equation from Part (a).

(c) Classify the reaction.


P a g e | 128

Experiment 10.
Empirical Formula

LEARNING OBJECTIVES
1. The students will be able to learn how the compounds are formed.
2. The students will be able to learn how the empirical formulas of the compounds are determined
experimentally.

BACKGROUND
The empirical formula of a compound is its simplest formula and can be determined experimentally.
The actual formula of a compound is known as the molecular formula of the compound. The
molecular and empirical formulas of a compound can be the same or different. For example, glycerin
has a formula of C3H8O3. Its empirical formula is also C3H8O3. However, C2H2 (acetylene) and C6H6
(benzene) have the sample empirical formula, CH. These two compounds have entirely different
physical and chemical properties. These two elements, C and H, can combine at different weight
ratios to form several compounds known as hydrocarbons.

Similarly, copper can combine with oxygen to form to different compounds, Cu2O, copper(I) oxide,
and CuO, copper(II) oxide, depending of the experimental conditions.

There are two parts in this experiment. In the first part, you will combine magnesium with oxygen, to
form magnesium oxide. In the second part, you will react magnesium with hydrochloric acid to form
magnesium chloride. Based on the experimental results, you will determine the empirical formula of
these two compounds.

The methods employed here are destructive to the compounds and require large quantities to work
with. New instrumental methods reveal the empirical and the molecular formulas of newly synthesized
compounds with much higher precision.
P a g e | 129

SAFETY INFORMATION.
1. No Eating or Drinking in the Laboratory.
2. Do not stare or look at the magnesium while it is burning in the crucible. It can be harmful.
3. Do NOT touch the crucible when it is HOT. Give ample time for the crucible to be cooled.
4. 6.0 M hydrochloric acid is CORROSIVE and can cause CHEMICAL BURNS.
5. Wash your hands before leaving the laboratory.

MATERIALS and CHEMICALS


Magnesium ribbon, 6 M HCl, crucible and cover, striker, tongs, wire gauze, evaporating dish, watch
glass, ring stand and O-ring, balance.
P a g e | 130

PRE-LABORATORY WORKED EXAMPLES.


1. A 0.500 g of Chromium, Cr, was heated and allowed to react with oxygen. The resulting product
has a mass of 0.962 g. Find the empirical formula for chromium oxide. Cr = 52; O = 16.

PLAN.
a. Find the mass or the mass percentage of each element.
b. Convert into moles.
c. Find the mole ratio by dividing all the numbers by the smallest mole number.

0.500
nChromium = = 0.009615 mol of Cr
52

(0.962  0.500)
nOxygen = = 0.02888 mol of O
16

0.009615
number of mole-atoms of Cr = =1
0.009615

0.02888
number of mole-atoms of O = =3
0.009615

The empirical formula of the chromium oxide is CrO3, chromium(VI) oxide.

2. A compound is composed of 5.6 g of iron, Fe, and 4.8 g of sulfur. What is the empirical formula of
the compound? Fe = 56 and S = 32.

5 .6
nIron = = 0.10 mol of Fe
56

4 .8
nSulfur = = 0.15 mol of S
32

0.10
number of mole-atoms of Fe = =1
0.10

0.15
number of mole-atoms of S = = 1.5
0.10

The empirical formula of the iron sulfide is Fe1S1.5. We always divide by the smaller number in order
to find the simplest whole number ratio:

The empirical formula of the compound is Fe2S3, iron(III) sulfide.


P a g e | 131
3. A compound has a following percent composition: 85.7% of C and 14.3% of H.
a. What is the empirical formula of the compound? C = 12 and H = 1
b. What is the molecular formula of the compound if the molecular weight of the compound is 140
g/mol?

85.7
nCarbon = = 7.14 mol of C
12

14.3
nHydrogen = = 14.3 mol of H
1

7.14
number of mole-atoms of C = =1
7.14

14.3
number of mole-atoms of H = =2
7.14

The empirical formula of the hydrocarbon is CH2.

b. The molecular formula is (mass of empirical formula)x = Molecular mass

(CH2)x = 140

14x = 140 ; therefore, x = 10.

The molecular formula of the compound is C10H20.


P a g e | 132

PRELAB QUESTIONS NAME: _____________________

1. At the end of the experiment will the final product weigh more or less than the starting material?

2. What accounts for the weight change in this experiment?

3. Calculate the percent composition by weight for the following:

a. VO3

b. V2O5

4. A sample of 2.5 grams of Vanadium was oxidized and the final product weighed 4.85 grams (2.35
grams of were oxygen). Determine the empirical formula for this VxOy compound. V = 50.9 and
O = 16.
P a g e | 133

PART A. Empirical Formula of Magnesium Oxide.


1. Weigh the empty crucible and the cover. Record its mass.

2. Place the crucible on the clay triangle as shown in the figure below. Light the Bunsen burner. Heat
the empty crucible and cover. Remove the heat. Cool and weigh the crucible and cover. Record the
mass.

Figure 1. Burning magnesium in a crucible.

3. Cut about 4 - 8 cm (or about 0.050 - 0.100 gram) of magnesium ribbon, roll it in a coil, and place it in
the crucible. Weigh the crucible, cover, and the Mg ribbon and record the mass.

4. Place the crucible in the clay triangle, light the Bunsen burner, and heat the crucible until the
magnesium catches fire. Now, using the tongs, place the cover on the crucible, and continue to heat
another 5-10 minutes. Slowly and carefully, using the tongs, tilt the cover to the side and keep heating
the crucible another 10 minutes. Now, you will observe a grayish solid in the crucible.

5. Remove the heat and let the crucible cool. Take the crucible down using wire gauze and tongs.
Weigh the crucible, cover, and the residue. This is the mass after the first heating. Record the mass.

6. Place the crucible in the clay triangle again, light the Bunsen burner, and reheat the crucible.
Remove the heat. Allow the crucible to cool and weigh the crucible, cover, and the residue again.
Record the mass. This is the mass after the second heating.

NOTE: The temperature of the Bunsen burner flame is about 900ᵒC. At this temperature, only the
oxygen will combine with magnesium. Much higher temperatures are needed in order for the nitrogen
in the air to combine with magnesium.

7. Clean the crucible and repeat the experiment with another sample of magnesium ribbon.
P a g e | 134

PART B. Empirical formula of Magnesium chloride.


1. Weigh an evaporating dish and record its mass empty.

2. Place about 4-cm of magnesium ribbon and weigh the evaporating dish and the magnesium.
Record the mass.

3. WEAR YOUR EYE GOGGLES AND MAKE SURE THE ARE NO OPEN FLAMES AROUND!
THE NEXT STEP SHOULD BE DONE IN A FUME HOOD.

Carefully, start adding 6 M HCl solution. The gas evolution (hydrogen) is intense. Keep adding the
acid slowly until all magnesium is dissolved.

4. Place the evaporating dish on a hot plate and cover it with a watch glass. Start heating the
contents to dryness. Stop the heat when the contents appear dry and no condensation is visible on
the watch glass. Cool the evaporating dish and weigh it. Record the mass of the evaporating dish and
the contents.

5. Repeat Step (4) to make sure that all the liquid is evaporated. Stop the heat and cool the
evaporating dish. Weigh it and record the mass after the second heating.
P a g e | 135
DATE: _____________ NAME: ________________________

DATA TABLE FOR PART A. USE A MILLIGRAM BALANCE FOR THIS EXPERIMENT.

TRIAL 1. TRIAL 2.
1. Mass of the empty crucible and cover. _________ g _________ g

2. Mass of the empty crucible and cover


after heating. _________ g _________ g

3. Mass of the crucible, cover, and Mg. _________ g _________ g

4. Mass of magnesium _________ g _________ g


(Line 3 – Line 2)

5. Mass of the crucible, cover, and Mg after


the first heating. _________ g _________ g

6. Mass of the crucible, cover, and Mg after


the second heating. _________ g _________ g

7. Mass of magnesium oxide _________ g _________ g


(Line 6 – Line 2)

8. Mass of oxygen combined _________ g _________ g


(Line 7 – Line 4)

9. Mole of magnesium _________ mol _________ mol


(Line 4) / (24.3)

10. Mole of oxygen _________ mol _________ mol


(Line 8) / (16.0)

11. Mole ratio of O to Mg. __________ __________


(Line 10) / (Line 9)

12. The empirical formula of the compound _______________


P a g e | 136
DATE: _____________ NAME: ________________________

DATA TABLE FOR PART B. USE A MILLIGRAM BALANCE FOR THIS EXPERIMENT.

TRIAL 1. TRIAL 2.

1. Mass of the evaporating dish empty. _________ g _________ g

2. Mass of the evaporating dish and magnesium _________ g _________ g

3. Mass of magnesium. __________ g _________ g


(line 2 – line 1)

4. Mass of the evaporating dish and product


after the first heating. _________ g _________ g

5. Mass of the evaporating dish and product


after the second heating. _________ g _________ g

6. Mass of magnesium chloride _________ g _________ g


(line 5 – line 1)

7. Mass of chlorine combined _________ g _________ g


(line 6 – line 3)

8. Mole of magnesium _________ mol _______ mol


(line 3) / (24.3)

9. Mole of chlorine _________ mol _______ mol


(Line 7) / (35.5)

11. Mole ratio of Cl to Mg. __________ _________

12. The empirical formula of the compound _______________


P a g e | 137

POSTLAB QUESTIONS NAME: __________________

1. A 0.334-gram sample of iron is reacted with excess oxygen to form an iron oxide. Calculate the
empirical formula of iron oxide if the mass of the product is 0.477 gram. Fe = 56; O =16.

2. A 0.450-gram sample of iron oxide was reduced to 0.315 gram of iron metal by reacting with
carbon. Determine the empirical formula of the iron oxide. Fe = 56; O = 16.
P a g e | 138
3. A compound is made of 0.14 gram of nitrogen and 3.81 grams of iodine. What is the empirical
formula of the compound? N = 14; I = 127.

4. A compound is made of C, H, and O. A 9.2 g sample of the compound contains 3.6 g of C and 0.8
g of H. What is the empirical formula of the compound? What is the molecular formula of the
compound if the molar mass of the compound is 92 g/mol? C = 12; H = 1; O = 16.

5. A compound is made of 30.4% of N and the rest is O. What is the empirical formula of the
compound? What is the molecular formula of the compound if the molar mass is 92.0 g/mol?
N =14; O = 16.
P a g e | 140

Experiment 11.

Percent Yield of a Chemical Reaction


LEARNING OBJECTIVES:

1. Students will be able to predict the amount of product produced from a double displacement
reaction and determine the percent yield of a chemical reaction.
2. Students will be able to identify the limiting and excess reactants.

BACKGROUND.

Stoichiometry is the mole to mole relationships between the reactants and the products of a chemical
reaction. Stoichiometric calculations are about the amounts of substances that react (reactants) and
form (products) in chemical reactions.

Chemical reactions are represented by equations which are balanced using the smallest whole-
number coefficients. A balanced chemical equation helps the chemist to calculate the amounts of
product formed in a chemical reaction. This is the theoretical yield of the reaction. In other words, it
is the maximum amount of product that can be obtained in a particular reaction from a known quantity
of reactants. This may not be always the case and the amount of product obtained in a chemical
reaction is usually less than the theoretical amount. The amount of product obtained in a chemical
reaction performed in the laboratory is called the actual yield. The percent yield of a reaction is then
given by the following expression:

Actual Yield
Percent yield = × 100
Theoritica l Yield

In this experiment, you will react Na2CO3 and CaCl2. You will prepare calcium carbonate, CaCO3,
from the reaction of aqueous solution of sodium carbonate, Na2CO3, with the aqueous solution of
calcium chloride, CaCl2. From the amounts of the reactants, you will determine which reactant is the
limiting reactant, and from this amount, will calculate the theoretical yield of calcium carbonate
produced. From the actual amount of calcium carbonate obtained, you can then calculate your
percent yield of reaction.
The balanced equation for this chemical reaction is:

CaCl2(aq) + Na2CO3(aq)  CaCO3(s) + 2NaCl(aq)

The net ionic equation of the reaction is:

Ca2+(aq) + CO32-(aq)  CaCO3(s)

(aq) and (s) mean aqueous and precipitate or solid, respectively.


P a g e | 141

SAFETY INSTRUCTIONS

1. Wear your eye goggles at all times.


2. All chemicals used in these experiments are generally recognized as safe. But, you MUST wash
your hands after you finish the experiment.
3. If your instructor recommends you use vacuum filtration, proceed with caution. The side arm of the
Erlenmeyer can break.

MATERIALS

Centigram balance, ring stand, O-ring, funnel, filter paper, clay triangle, spatula, stirring rod, 50-mL
graduated cylinder, two 150-mL beakers, watch glass, pencil, solid sodium carbonate, 0.50 M sodium
carbonate solution.
P a g e | 142

PRE-LABORATORY WORKED EXAMPLE.


A student adds 30.0 mL of 0.10 M potassium chromate into a beaker containing 100.0 mL 0.010 M
silver nitrate solution. The precipitate is filtered, washed and dried. The mass of the solid product is
0.15 gram. Cr = 52; Ag = 108; O = 16; N = 14; K = 39.
Note: The molarity of a solution is the number of moles of solute in 1.0 Liter of solution.
Note: The number of moles of solute is, mole = Molarity × Volume

a. Write the balanced equation representing the chemical reaction.

2AgNO3(aq) + K2CrO4(aq)  Ag2CrO4(s) + 2KNO3(aq)

b. Write the net ionic equation of the chemical reaction.

2Ag+(aq) + CrO42-(aq)  Ag2CrO4(s)

c. Determine the number of moles of each reactant.

nAgNO3 = M × V = 0.010 mol.L-1 × 0.100 L = 0.0010 mol AgNO3

nK2CrO4 = M × V = 0.10 mol.L-1 × 0.030 L = 0.0030 mol K2CrO4

d. Identify the limiting reactant. Note: Lesser amount is the limiting reactant.

1 mol Ag 2 CrO4 332 g Ag 2 CrO4


0.0010 mol AgNO3 × × = 0.166 g Ag2CrO4
2 mol AgNO3 1 mol Ag 2 CrO4

1 mol Ag 2 CrO4 332 g Ag 2 CrO4


0.0030 mol K2CrO4 × × = 0.996 g Ag2CrO4
1 mol K 2 CrO4 1 mol Ag 2 CrO4

The limiting reactant is AgNO3.

e. Determine the theoretical yield of the reaction.

The theoretical yield of the reaction is 0.166 g.

f. Calculate the percent yield of the reaction.

Actual Yield 0.15


Percent yield = × 100 = × 100 = 88%
Theoritica l Yield 0.17
P a g e | 143

PRE-LABORATORY QUESTION NAME: ____________________

A student weighs 1.00 gram of zinc powder and places it into a beaker containing 25.00 mL of a
0.50 M CuCl2 solution. After the reaction is completed, the solid (metallic copper) is recovered from
the bottom of the beaker, filtered, washed with hydrochloric acid solution, and dried completely. The
mass of the precipitate is 0.72 gram. Zn = 65, Cu = 63.5C; Cl = 35.5.

1. Write the equation representing the chemical reaction.

2. Determine the number of moles of zinc reactant present.

3. Determine the number of moles of CuCl2 reactant present

4. Calculate the theoretical yield of the reaction.

5. Calculate the percent yield of this reaction.


P a g e | 144

EXPERIMENTAL PROCEDURE

1. Using a pencil, write your name on the edges of the filter paper, and place it on a watch glass.
Weigh the filter paper and the watch glass together.

2. Weigh an empty 150-mL beaker and record its mass.

3. Place about 1.00 gram of the powdered sodium carbonate, Na 2CO3, into the beaker, and weigh it
again. Record the mass.

4. Dissolve the solid using about 25 to 30 mL of deionized water.

5. Using the 50-mL graduated cylinder, measure 50.0 mL of 0.50 M CaCl2 solution.

6. Add the CaCl2 solution into the beaker containing the Na2CO3 solution. Observe the precipitation
reaction. Mix the mixture with a stirring rod.

7. Assemble the stand, O-ring, funnel, and filter paper according to the figure described below. Filter
the mixture.
P a g e | 145
8. Wash the residue twice with 5.0 mL of deionized water. Rinse with isopropyl alcohol and dry in the
oven for half an hour.

9. Carefully remove the filter paper and precipitate and place it on the watch glass.

10. Give the watch glass to your instructor.

11. Weigh the filter paper, dried residue, and watch glass. Record the mass.

12. Calculate the percent yield of the reaction.


P a g e | 146
DATE: ___________ NAME: _________________________

DATA SHEET FOR EXPERIMENT 11.

TRIAL 1 TRIAL 2

1. Mass of the filter paper and the watch glass: __________ g _________ g

2. Mass of the beaker: __________ g _________ g

3. Mass of the beaker + Na2CO3: __________ g _________ g

4. Mass of the Na2CO3: __________ g _________ g


(line 3) – (line 2)

5. Mass of the dried residue, watch glass, and __________ g _________ g


filter paper:

6. Mass of the CaCO3 (residue): __________ g _________ g


Actual Yield: (line 5) – (line 1)

7. Theoretical yield# of CaCO3: __________ g _________ g


(From stoichiometric calculations)

8. Percent Yield of the reaction: __________ % _________%


[(line 6) / (line 7)] × 100

8. Average percent yield: _____________ %

#Use the following stoichiometric calculations to determine the theoretical yield.

1mol Na 2 CO3 1mol CaCO3 100 grams


_____ grams of Na 2 CO3     grams CaCO3
106 grams 1mol Na 2 CO3 1mol CaCO3

1mol CaCO3 100 grams


_____ mol of CaCl2    grams CaCO3
1mol CaCl2 1mol CaCO3
P a g e | 147

POSTLAB QUESTIONS NAME: ________________________

1. A sample of 30.00 grams of potassium phosphate was added to a solution containing 100.0 mL of
0.25 M calcium nitrate. Calcium phosphate was precipitated according to the following reaction.

2K3PO4(s) + 3Ca(NO3)2(aq)  Ca3(PO4)2(s) + 6KNO3(aq)

2.25 grams of Calcium phosphate were recovered. Calculate the theoretical yield and the percent
yield.

2. A student adds 50.0 mL of 0.25 M K2SO4 solution to 25.0 mL 0.50 M BaCl2 solution. What is the
mass of the precipitate that can be recovered if the percent yield of the reaction is 75%? Ba = 137;
Cl = 35.5; S = 32 O = 16; K = 39. Note: You must write the balanced equation first.

BaCl2(aq) + K2SO4(aq)  BaSO4(s) + 2KCl(aq)


P a g e | 148
3. A 4.00-gram sample of sodium chloride was added to a beaker containing 40.0 mL 0.033 M lead(II)
nitrate solution. Lead(II) chloride was precipitated according to the following reaction.

2NaCl(s) + Pb(NO3)2 (aq)  PbCl2(s) + 2NaNO3(aq)

Calculate the mass of lead(II) chloride that can be recovered. Pb = 207; Na = 23; Cl = 35.5; N = 14;
O = 16.

4. A student adds 50.0 mL of 0.25 M KCl solution to 25.0 mL 0.50 M AgNO3 solution. What is the
mass of the precipitate that can be recovered if the percent yield of the reaction is 90%? Ag = 108;
Cl = 35.5; N = 14; O = 16; K = 39. Note: You must write the balanced equation first.
P a g e | 150

Experiment 12.
Paper Chromatography
LEARNING OBJECTIVES
Students will be able to:

1. Learn the theoretical and practical concepts of paper chromatography.


2. Apply the concept of intermolecular forces to separation processes.
3. Use the process of paper chromatography to separate various mixtures of substances in a
qualitative way.

BACKGROUND
A. USES

Paper chromatography is an analytical tool used in forensic science and organic chemistry laboratory.
This technique was first introduced by Tyndall when he experimented with separating pigments from
plants. Chemists use this technique for separating and identifying mixtures that are or can be colored,
especially inks and food colorings. This method has been largely replaced by TLC, thin layer
chromatography, where a glass plate is coated with thin layer of alumina, Al2O3. Meanwhile, paper
chromatography is still a powerful teaching tool. Two-dimensional chromatography involves using two
solvents and rotating the paper 90° in between solvent changes. This enhances the separation of the
mixture. This technique can also be applied to colorless mixtures such as amino acids and
analgesics. The spots can be developed under a UV lamp, where they are seen as violet spots. The
spots can also be colored by combining them with other chemicals (derivatization) such as iodine
vapor.

B. THEORY

A small sample of a mixture is placed on porous paper which is in contact with the solvent. The
solvent moves through the paper due to capillary action and dissolves the mixture spot. The
components of the sample start to move along the paper in the same direction as the solvent. If the
solvent or solvent mixture is polar, the polar constituents of the mixture will move along with the
solvent (mobile phase). The others will be retained longer and move at a slower rate.

Components of the mixture with a stronger attraction to the paper than to the solvent will move more
slowly than the components with a strong attraction to the solvent. The differences in the rates with
which the components travel along the paper leads to their separation. Particular mixtures will have
chromatographic patterns that are consistent and reproducible as long as the paper, solvent,
temperature, and time are constant. Different solvents will change the patterns of component
separation.
P a g e | 151

Quantitative analysis of chromatographic data

Retention factors, Rf, are calculated to give a quantitative measure of a component’s properties in a
mixture.

When chromatograms are made with the same solvent and developed for the same amount of time,
Rf values are identical for the same components in the mixture.

Paper chromatography presents some advantages and disadvantages. Paper chromatography is


cheap, requires very small amount of sample size, and quick to perform and easy to learn. The
sample can also be recovered after the experiment. With a correctly chosen mobile phase
(chromatographic solvent), an analyst can rapidly determine the number of constituents in the
sample. It has its limitations. It is an analytical tool to identify the components of a mixture, not to
separate compounds at large scales. A better preparative tool, such as ultra HPLC, is employed by
pharmaceutical companies to purify the active compound present in a complex matrix.
P a g e | 152

MATERIALS

400-mL beakers, cut filter papers, pencil, toothpicks, ruler, Al-foil.

CHEMICALS

For Procedure A. A 5% salt solution and a 50:50 acetone:water mixture.


Water soluble food colors (red, yellow, green, and blue), pencil, toothpicks, ruler,

For Procedure B. 50:50 hexane:ethylacetate mixture, 0.1% solutions of acetyl salicylic acid (aspirin),
ibuprofen, acetaminophen, caffeine, and Excedrin in isopropyl alcohol.

Optional Excedrin® tablets and isopropyl alcohol.

SAFETY
1. No eating and drinking in the laboratory. Obtain and wear eye goggles at all times.
2. Some of the solvents mixtures contain harmful solvents for your contact lenses. Perform the elution
process in the fume hoods.
3. After you finish the experiment, clean up and wash your hands.
P a g e | 153

PART A. PAPER CHROMATOGRAPHY OF FOOD DYES

PROCEDURE A-1.
In a fume hood, using a Pasteur pipet, place one drop of each of the mobile phases on the
workbench and allow them to evaporate. Identify the faster and slower evaporating solvents and label
them as “the more polar” and “the less polar”, respectively.

PROCEDURE A-2.
1.Obtain two 400-mL beakers. Add 15-20 mL of mobile phase A to one and 15-20 ml of mobile phase
B to the other. Cover each beaker with aluminum foil. This will help saturate the headspaces with
solvent vapors and run the chromatograms faster. The solvent choices are:

A. 5% aqueous NaCl solution.

B. 50:50 (volume by volume) acetone: water mixture.

2. Cut two 10 cm long by 5 cm wide pieces of filter paper to use as stationary phases.

3. With a pencil, draw two lines: one 2 cm from the bottom (solvent front) and the other 2 cm from the
top (solvent end) of the filter paper.

Figure 1. Schematic for paper chromatography.

R: red food color Y: yellow food color G: green food color


B: blue food color S: chosen sample
P a g e | 154
4. Using toothpicks or Pasteur pipets, make small spots of food dyes on the bottom line. Label them
as R (red), Y (yellow), G (green), and B (blue). Also make a spot using your pen (sample) and label it
as S. Let them dry.

Figure 2. Spotting the samples on paper.

5. Make a second spot of the appropriate food dye or ink on top of the original spots. This will double
the concentration of test material without increasing the size of the spots.

6. Insert a filter paper into each beaker, secure it by hanging it from wooden splints, and wait until the
solvent reaches the top line.

7. Remove the filter papers from the beaker, place them in the fume hoods and allow some time for
them to dry.

8. Using a centimeter ruler measure the distances (from the spotting line to the centers of the spots)
traveled by the dyes.

Note: A group of students can use 5% NaCl as a mobile phase and another group can use the
(50:50) acetone:water mixture as the mobile phase. Afterwards, the students can share their data.
P a g e | 155

PROCEDURE A-3. Interpreting the Data.


The Rf value for each spot should be calculated. Known Rf values can be compared to those of
unknown substances to aid in their identifications.

Figure 3. Elution profile of food coloring using 5% NaCl solution as a mobile phase.
P a g e | 156

PART B. PAPER CHROMATOGRAPHY OF ANALGESICS

PROCEDURE B-1. OPTIONAL or Thin Layer Chromatography.


https://www.youtube.com/watch?v=qdmKGskCyh8

THE EXPERIMENT SHOULD BE PERFORMED IN A FUME HOOD.

1. Into a 400-mL beaker, add about 15-20 mL of a 50:50 (volume / volume) hexane:ethyl acetate
mixture and cover it with aluminum foil.

2. Cut a 10 cm long by 5 cm wide piece of filter paper to use a stationary phase.

3. With a pencil, draw two lines: one 2 cm from the bottom (solvent front) and the other 2 cm from the
top (solvent end) of the filter paper.

4. Using toothpicks, make small spots of analgesics on the bottom line. Label them, with a pencil, as
A (acetyl salicylic acid, aspirin), I (ibuprofen), Ac (acetaminophen), C (caffeine) and E (Excedrin®).
Let them dry.

5. Insert the filter paper in the beaker, secure it by hanging it from wooden splints, and wait until the
solvent reaches the top line.

6. Remove the filter paper from the beaker, place it in the fume hood, and allow some time for it to
dry.

7. Under the UV lamp, observe the spots. Outline them in pencil. Using a cm ruler measure the
distances traveled by the compounds.

8. Identify the ingredients present in Excedrin®.

9. Circle the ingredients below that are present in the Excedrin sample:

RESULT:

Acetyl salicylic acid Ibuprofen Acetaminophen Caffeine


P a g e | 157
DATE: _________ NAME________________________

Paper Chromatography Part A


Sample Color Distance the Distance the Mobile Rf
Sample Traveled Phase Traveled

Paper Chromatography Part B


Sample Color Distance the Distance the Mobile Rf
Sample Traveled Phase Traveled
P a g e | 158
POSTLAB QUESTIONS NAME: __________________________________
1. What is the purpose of double-spotting the chromatography paper at the beginning of the
experiment?

2. Define the mobile and stationary phases in chromatography.

3. Which of the solvents or solvent mixtures is the less polar? The more polar? What makes a
substance polar or nonpolar?

4. There are two substances, Q and T, in a mixture. Substance Q is more polar than substance T.
Which one will have a larger Rf when pure water is used as an elution phase? Why?
P a g e | 159
5. You identified the most and the least traveled dyes, and the most and least polar solvents. What
causes the components to separate? Based on these observations and using the concept of “LIKE
DISSOLVES LIKE” identify:

a. Which of the dyes is the most polar? Why?

b. The least polar? Why?

6. How many components were in the dye mixture that you used? Did the manufacturer of the pen
use a single color ink or a mixture of different color inks?

7. Name two practical uses for paper chromatography.

8. An unknown liquid sample is analyzed using paper chromatography using solvent A as the mobile
phase. One spot is observed after the plate is developed and visualized. The same unknown
substance is re-analyzed using solvent B as the mobile phase. This time, two spots are observed
after the plate is developed and visualized. Is the unknown sample a pure substance or a mixture?
Explain.
P a g e | 160

Experiment 13.
Oxidation-Reduction
LEARNING OBJECTIVES
The students will be able to
1. Learn how to calculate the oxidation state of the elements in compounds.
2. Learn activity series of the metals.
3. Balance redox reactions.

BACKGROUND
Some elements, such as gold, exist in elemental states in Nature. Most of the elements exist
combined with other elements in form of compounds. Some elements are even found in more than
one oxidation state, e.g. iron(II) oxide, FeO, iron(III) oxide, Fe 2O3, or magnetite, Fe3O4.

Compounds are formed from at least two elements. The elements are combined by either sharing or
exchanging their valence electrons. Metals tend to lose electrons. Their oxidation states are positive
integers. The nonmetals tend to gain electrons. Their oxidation states are negative integers.
The alkali and alkaline earth metals have +1 and +2 oxidation states, respectively. The representative
element metals in column 13 (IIIA) have oxidation states of +3 with the exception of thallium which
can have +1 and +3 oxidation states.

The oxidation state of fluorine is ALWAYS -1. It is the most electronegative element. No other
element can make fluorine to have a positive oxidation number. The second most electronegative
element is oxygen. It has an oxidation state of -2 in most compounds. However, combined with
fluorine, as in OF2, it has a +2 oxidation state. In H2O2, hydrogen peroxide, oxygen has an oxidation
state of -1.

Nitrogen’s oxidation state, when combined with less electronegative elements, is -3. But, when
nitrogen is combined to fluorine and oxygen, it has positive oxidation states. This is also true for
chlorine, which will have an oxidation state of -1 when combined with less electronegative elements.
In prelaboratory worked problems, you will learn how to calculate the oxidation states of each of the
elements present in various compounds.

There are five different experiments in laboratory session. After finishing this laboratory, you will be
able to identify reactions in which electron transfers occur. These reactions are known as redox
reactions.
P a g e | 161

SAFETY PRECAUTIONS

1. No drinking or eating in the laboratory. You have follow all laboratory safety guidelines.
2. Obtain and wear chemical-splash eye goggles at all times.
3. Dilute acids and bases are irritating to eyes and skin.
4. After the experiment is finished, clean up and wash hands thoroughly with soap and water before
leaving the laboratory.
5. 6 M is corrosive. Handle with care.

MATERIALS and CHEMICALS

Aluminum wire Potassium permanganate, KMnO4, 0.20 M


Magnesium ribbon Sulfuric acid, H2SO4, 4.0 M
Copper, metal Potassium iodide, KI, 0.10 M
Zinc, metal Commercial Clorox® solution
Silver nitrate, 0.10 M Mineral oil.
AgNO3, 0.10 M Small and large test tubes
Magnesium nitrate, Mg(NO3)2, 0.10 M Test tube racks
Copper(II) nitrate, Cu(NO3)2, 0.10 M Beaker, 50- and 150-mL
Zinc nitrate, Zn(NO3)2, Plastic droppers
Nitric acid, HNO3, 6.0 M Hot plates.
Hydrogen peroxide, 3%, H2O2,

NOTE: Dispose of all chemicals according to Federal, State, and Local Government Rules and
Regulations. Ask your instructor before dumping any of these chemicals into the sinks.
P a g e | 162

PRE-LABORATORY WORKED EXAMPLES.


1. What is the oxidation state of nitrogen in NF3, nitrogen trifluoride?

Solution. This is a neutral compound. The sum of the oxidation states must be equal to zero. We
know that F has an oxidation state of -1.

N + 3 (F) = 0

N + 3 (-1) = 0

N - 3 = 0 and N = +3.

The oxidation state of N is +3.

2. What is the oxidation state of manganese in KMnO4, potassium permanganate?

Solution. Again, this is a neutral compound. The sum of the oxidation states must be equal to zero.
We know that K has an oxidation state of +1 and O has an oxidation state of -2 (except in two
compounds given in BACKGROUND information).
.

K + Mn + 4 (O) = 0

1 + Mn + 4 (-2) = 0

1 + Mn – 8 = 0

Mn - 7 = 0 and Mn = +7.

The oxidation state of Mn is +7.

3. What is the oxidation state of chromium in Cr2O72-, the dichromate ion?

Solution. This is a negatively charged ion. The sum of the oxidation states must be equal to -2. We
know that O has an oxidation state of -2.

2 (Cr) + 7 (O) = -2

2 Cr + 7 (-2) = -2

2 Cr - 14 = -2 and Cr = +6.

The oxidation state of Cr is +6.


P a g e | 163
4. 4. A piece of metallic zinc is placed in a copper(II) solution. Zinc metal dissolves and copper metal
deposits. Balance the following redox reaction in aqueous solution:

Cu2+(aq) + Zn(s)  Cu(s) + Zn2+(aq)

Solution. Zn metal is losing two electrons. Its oxidation state changes from 0 to +2. Losing electrons is
called oxidation. The Cu2+ ions are gaining two electrons. Their oxidation state changes from +2 to 0
(elemental state). This process of gaining electrons is called reduction.

Zn  Zn2+ + 2e- (Oxidation)


Cu2+ + 2e-  Cu (Reduction)

Overall reaction Cu2+ + Zn  Cu + Zn2+

The overall reaction is balanced. There are same number of atoms and charges on either sides of the
reaction.

5. Metallic aluminum is immersed into an aqueous solution of silver nitrate. Aluminum metal dissolves
and metallic silver deposits. Balance the following net ionic equation.

Ag+(aq) + Al(s)  Ag(s) + Al3+(aq)

Solution. Al metal is losing three electrons. Its oxidation state changes from 0 to +3. Losing electrons
is called oxidation. The Ag+ ions are gaining one electron. Their oxidation state changes from +1 to 0
(elemental state). This process of gaining electrons is called reduction.

Al  Al3+ +3e- (Oxidation)


Ag+ + 1e-  Ag (Reduction)

Here we need another step. The number of electrons gained or lost must be equal. The three
electrons will be gained by three silver ions. Therefore, the second equation will be multiplied by 3.

Al  Al3+ +3e- (Oxidation)


3Ag+ + 3e-  3Ag (Reduction)

Overall reaction 3Ag+ + Al  3Ag + Al3+

The overall reaction is balanced. There are same number of atoms and charges on each side of the
reaction.
P a g e | 164

PRELAB QUESTIONS NAME: ___________________


1. Identify two oxidation states of iron.

2. Provide a molecule of oxygen which has the following oxidation states.

a. 0 ____________

b. -2 ____________

c. -1 ____________

d. +2 ____________

3. Provide appropriate examples for nitrogen.

a. A positive oxidation number ____________

b. A negative oxidation number ____________

4. An example of an element that naturally exists with a zero oxidation number.


P a g e | 165

EXPERIMENTAL PROCEDURES

DATE: _______________ NAME: _________________________

PART A. OXIDATION OF METALLIC COPPER WITH NITRIC ACID.


(Note: THIS EXPERIMENT MUST BE PERFORMED IN A FUME HOOD. DO NOT BREATH THIS
GAS EVOLVED!).

1. In a large test tube, place a small piece of metallic copper.

2. Carefully add 10 drops of 6.0 M HNO3 solution.

3. Record your three observations below.

a. ______________________________________________________.

b. ______________________________________________________.

c. ______________________________________________________.

4. Balance the following redox reaction.

Cu(s) + H+(aq) + NO3-(aq)  Cu2+(aq) + NO2(g) + H2O(l)


P a g e | 166
DATE: _______________ NAME: _____________________________

PART B. REDUCTION OF COPPER IONS BY METALLIC ALUMINUM

1. Fill a 250-mL beaker with water and start heating it on a hot plate.

2. Place about 2 mL of 0.10 M copper sulfate solution, CuSO4, in a large test tube.

3. Place the test tube in the beaker. Continue heating.

4. Obtain a 4 cm long piece of aluminum wire. Coil it. Place it in the test tube.

5. Observe the reaction for about 10 minutes. Record your observations below.

a. ______________________________________________________.

b. ______________________________________________________.

c. ______________________________________________________.

6. Balance the following equation as described in prelaboratory example #5.

Cu2+(aq) + Al(s)  Cu(s) + Al3+(aq)


P a g e | 167
DATE: _______________ NAME: _____________________________

PART C. REACTION OF PEROXIDE WITH ACIDIFIED POTASSIUM


PERMANGANATE SOLUTION.
THIS EXPERIMENT MUST BE PERFORMED IN FUME HOOD!

1. In a large test tube, add 10 drops of 0.20 M potassium permanganate solution.

2. Add 5 drops of 4.0 M sulfuric acid solution to the test tube.

3. Using a plastic Beral pipet, add 1 mL of 3% commercial hydrogen peroxide solution to the test
tube.

4. Light a wooden splint and slowly (CAREFULLY) bring it to the mouth of the test tube.

5. Record your observations below.

a. ______________________________________________________.

b. ______________________________________________________.

c. ______________________________________________________.

6. Balance the corresponding net ionic equation of the reaction given below.

H2O2(aq) + H+(aq) + MnO4-(aq)  O2(g) + H2O(l) + Mn2+(aq)


P a g e | 168
DATE: _______________ NAME: ____________________________

PART D. REACTION OF A SOLUTION OF POTASSIUM IODIDE WITH A


HYPOCHLORITE (Clorox®) SOLUTION.
THIS EXPERIMENT MUST BE PERFORMED IN FUME HOOD!

1. In a large test tube, add 10 drops of 0.20 M potassium iodide solution.

2. Add 5 drops of Clorox® to the test tube. Carefully swirl the test tube.

3. Add 10 drops of mineral oil to the test tube. Carefully shake the test tube.

4. Record your observations below.

a. ______________________________________________________.

b. ______________________________________________________.

c. ______________________________________________________.

REMOVE THE MINERAL OIL LAYER WITH A BERAL PIPET and PLACE IT IN THE WASTE
CONTAINER.

5. Balance the corresponding net ionic equation of the reaction given below.

I-(aq) + Cl2(g)  I2(s) + Cl-(aq)


P a g e | 169
DATE: _______________ NAME: _____________________________

PART E. SINGLE REPLACEMENT REACTIONS. METAL REACTIVITIES.

PROCEDURE:

1. Obtain a test tube rack and 8 small test tubes.

2. Add a few mL of silver nitrate to test tubes 1 and 3.

3. Add a few mL of copper nitrate to test tubes 2 and 4.

4. Add a few mL of sulfuric acid to test tubes 5 and 6.

5. Add a few mL of zinc nitrate to test tube 7.

6. Add a few mL of magnesium nitrate to test tube 8.

7. Add a small piece of copper to test tubes 1 and 2.

8. Add a small piece of zinc to test tubes 3, 4, and 5.

9. Add a small piece of magnesium in test tubes 6 and 7.

10. Add a small piece of zinc to the test tube 8.


P a g e | 170
DATE: ________________ NAME: _______________________________

DATA TABLE FOR PART E.


Place the letter R in the table below if you observe a reaction and NR if no reaction is observed.

Test tube Solution Copper Zinc Magnesium


Silver nitrate
1 (AgNO3)

2 Copper(II) nitrate
Cu(NO3)2

3 Silver nitrate
(AgNO3)

4 Copper(II) nitrate
Cu(NO3)2

5 Sulfuric acid (H2SO4)

6 Sulfuric Acid (H2SO4)

7 Zinc nitrate
Zn(NO3)2

8 Magnesium nitrate
Mg(NO3)2

4. Record your observations below.

Test tubes 1 and 2. ______________________________________________________

______________________________________________________________________

Test tubes 3, 4 and 5. ____________________________________________________

______________________________________________________________________.

Test tubes 6 and 7. ______________________________________________________

Test tube 8. ____________________________________________________________


P a g e | 171
5. First, write the chemical equation and then, if applicable, write the net ionic equation for the
reaction that took place in each test tube.

Test tube 1.

Test tube 2.

Test tube 3.

Test tube 4.

Test tube 5.

Test tube 6.

Test tube 7.

Test tube 8.
P a g e | 172

POSTLAB QUESTIONS NAME: ____________________

1. Determine the oxidation states of the bold-faced elements in each of the following compounds.

a. K2Cr2O7

b. NaClO4

c. H2SO3

d. Fe2O3

e. Mg(NO3)2

f. Ba(IO3)2

g. H3PO4

2. Identify the type of each of these half-equations and balance them.

a. Mg  Mg2+

b. H+  H2

c. NO3-  NO

d. SO2  SO3
P a g e | 173
e. I-  IO3-

f. SO42-  SO2

g. MnO4-  Mn2+

3. A mixture of zinc metal and powdered sulfur is added to an evaporating dish. The mixture is ignited
with a very hot copper wire. A solid with different chemical properties is produced and white sulfur
dioxide gas is also produced. Balance the following equations (that represent the two consecutive
reactions described) below.

Zn(s) + S(s)  ZnS(s)

ZnS(s) + O2(g)  ZnO(s) + SO2(g)


P a g e | 175

Experiment 14.
Solutions and pH
LEARNING OBJECTIVES
The students will be able to:
1. Learn how to prepare aqueous solutions.
2. Learn to dilute solutions.
3. Measure the pH of a solution.

BACKGROUND
Solutions are homogeneous mixtures. Solutions have two parts: the solute (the dissolved substance)
and the solvent (the one that dissolves the solute). In aqueous solutions, the solvent is distilled or
deionized water. Tap water cannot be used because it contains dissolved solutes. Pure water does
not conduct electricity; it is a non-electrolyte. Water has high dielectric constant to break the
electrostatic interactions between ions with opposite charges. In solution, the ions can travel freely
and they can conduct electricity. The types of solutes, where all the dissolved molecules are fully
dissociated, are called strong electrolytes. Solutions can usually be prepared by dissolving a solid
substance in water. The quantitative description of a solution is expressed by molarity. The molarity of
a solution is the number of moles of solute in a liter of solution.

M=

M = molarity; n = number of moles; V = volume in liter

Example 1: A solution contains 11.70 g of Sodium Chloride in 250.0 mL of solution. What is the
molarity of the solution? Na = 23; Cl = 35.5
Solution:
NaCl = 58.5 g/mol and V = 0.2500 L

n= 0.200 mol/L.

M= 0.800 mol/L.

Example 2: A sample of 4.90 g of CuSO4.5H2O is dissolved in enough water to prepare 400.0 mL


solution. What is the molarity of the solution? Cu = 63.5; S = 32; O =16; H = 1

Solution: The formula weight of is CuSO4.5H2O = 249.5 g/mol.

n= 0.0196 mol.

M= 0.0491 mol/L.
P a g e | 176
Sometimes, a solution can be highly concentrated (high solute content) and needs to be diluted. The
amount of solute remains the same, however by adding more solvent the concentration of the
solution will be lowered. The formula used to calculate the new concentration is:

Mi × Vi = Mf × Vf

Note: i for initial and f for final.

Some substances, such as HCl and HNO3, when dissolved in water release H+ ions into solutions.
These solutions are acidic and have a pH < 7.0. Others, such as NaOH and KOH, release OH - ions
into solutions. These solutions are basic and have a pH >7.0. Solutions that have a pH = 7.0 are
called neutral. A neutralization reaction is a reaction between an acid and a base to produce a salt
and water.

In summary, the pH of a solution can vary as follows:

Acidic: 0 pH < 7.0 Neutral: 7.0 Basic: 7.0 < pH 14.0

Example 3: What volume of water is needed to dilute 50.0 mL 12 M HCl solution in order to obtain 3
M HCl solution?

Solution: (12 M) × (0.050 L) = (3 M) × (Vf)

0.60 = (3 M) × (Vf)

Vf = 0.200 L = 200 mL.

The volume of water needed is 200 - 50 = 150 mL

There are two parts in this laboratory experiment. In the first part, you will prepare 100.0 mL of a 0.10
M solution of copper(II) sulfate using solid CuSO4.5H2O. Then, you will prepare more solutions by
diluting this stock solution. In the second part, you will prepare solutions of acids and bases and
determine their pH’s.
P a g e | 177

SAFETY PRECAUTIONS

1. No drinking or eating in the laboratory. You have follow all laboratory safety guidelines.
2. Obtain and wear chemical-splash eye goggles at all times.
3. Dilute acids and bases are irritating to eyes and skin.
4. After the experiment is finished, clean up and wash hands thoroughly with soap and water
beforeleaving the laboratory.

MATERIALS

NaCl solution, 0.10 M Toothpicks


CuSO4.5H2O, solid Volumetric flasks 100-mL (4)
HCl and NaOH, 0.10 M Small test tubes (5)
Phenolphthalein Plastic pipets
Deionized water pH paper
Balance Beakers 50-mL
Well-plates Graduated cylinders, 10 and 25-mL
P a g e | 178

PRELAB QUESTIONS NAME: ____________________


1. Define the units for molarity.

2. A solution contains 5.25 grams of sodium chloride in 750.0 mL of solution. Determine the molarity
of the solution. Na = 23, Cl = 35.5

3. A 7.59 gram of sample of CuSO4.5H2O is dissolved in enough water to prepare 300.0 mL solution.
Determine the molarity of the solution. Cu = 63.5, S = 32., O = 16., H = 1.

4. What volume of water is needed to dilute of a 10.0 mL 5.0 M sugar solution to obtain a 0.25 M
sugar solution?
P a g e | 179

DATE: ____________ NAME: ________________________

DATA SHEET FOR PROCEDURE A.

Part A. Preparation of stock solution and PDNLQJserial dilutions

PROCEDURE

1. Weigh a 50-mL beaker. Record the mass. ________ g

2. Add about 0.5 gUDP of CuSO4.5H2O. Record the mass. ________ g

3. Mass of CuSO4.5H2O
(/ine 2- /ine 1)  ________ g

4. Determine the number of mol of CuSO4.5H2O.


((/ine 3) / 249.5JPRO) ________ mol

5. Add some deionized water to the beaker. Carefully mix with a stirring rod and transfer its contents
to a 100mL volumetric flask #1.

6. Repeat step 5 until all solid is dissolved. Then fill the volumetric flask to the mark with deionized
water (100.0mL = 0.100 L).

7. Calculate the molar concentration.


(line 4 / 0.100) ________ mol/L

8. Pipette 10.00 mL of solution from volumetric flask #1 and transfer it to volumetric flask #2 Fill to the
mark with deionized water. Determine the molarity of the new solution.
________ M.

9. Pipette 10.00 mL of solution from volumetric flask #2 and transfer it to volumetric flask #3. Fill
volumetric flask #3 to the mark with deionized water. Determine the molarity of the new solution.

________ M

10. Pipette 10mL of solution from volumetric flask #3 and transfer it to volumetric flask #4. Fill
volumetric flask #4 to the mark with deionized water. Determine the molarity of the new solution.

________ M
P a g e | 180
11. Obtain four small test tubes. Arrange them in a row. Fill each test tube, in turn, with the solution
from volumetric flask 1, 2, 3, and 4.

12. Take a picture of the test tubes with your smartphone. (BE CAREFUL TO NOT WET YOUR
PHONE!)

12. Obtain the CuSO4 solution with an unknown concentration. Compare the color visually or to the
picture taken with your phone and evaluate its concentration.

The concentration of the unknown solution is _______ M


P a g e | 181
DATE: ____________ NAME: ________________________

PART B.

THE pH’s OF ACIDS AND BASES

PROCEDURE$.

1. Obtain a well-plate. Add 20 drops of 0.10 M HCl to the first well. Add 20 drops of 0.10 M
NaOH to the second well and 20 drops of 0.10 M NaCl solution into the third well.

2. 8VLQJDS+VWLFNPeasure the pH of each solution and record them.

A. pH of 0.10 M HCl solution ________.


B. pH of 0.10 M NaOH solution ________.
C. pH of 0.10 M NaCl solution ________.

3. Add one drop of phenolphthalein indicator to each of the wells and record the colors.

A. Color of 0.10 M HCl solution _________.


B. Color of 0.10 M NaOH solution _________.
C. Color of 0.10 M NaCl solution _________.

352&('85(%

Obtain a small test tube and using a plastic pipet add 5 drops of 0.10 M of HCl and 1 drop of
phenolphthalein indicator solution. Add 10 drops of 0.10 M of HCl and 1 drop of phenolphthalein to a
second small test tube. Add 15 drops of 0.10 M of HCl and 1 drop of phenolphthalein to a third small
test tube. The total drops of acid is 30 drops.

Using a disposable pipette, to each tube, add 0.10 M NaOH solution until it turns pink. Record
the number of drops of NaOH for each:

Test tube 1. __________ drops of HCl __________ drops of NaOH


Test tube 2. __________ drops of HCl __________ drops of NaOH
Test tube 3. __________ drops of HCl __________ drops of NaOH

Total 30 drops of HCl __________ drops of NaOH

Compare the total number of drops of NaOH to the total number of drops (30 drops) of HCl.

 Actual values should be equal. Explain.



P a g e | 182

POSTLAB QUESTIONS NAME: _______________________

1. In the experiment PART B did you achieve neutralization with 30 drops of 0.10 M NaOH solution?
If not, explain why.

2. What mass of K2SO4 is needed to prepare 500.0 mL of a 0.75 M K2SO4 solution?


K = 39; S = 32; O =16.

3. What volume of water is needed to dilute a 25 mL of a 5.0 M sugar solution to obtain a 1.0 M
sugar solution?
P a g e | 183
4. 50.00 mL of a 0.25 M NaCl solution is added to 150.00 mL of a 0.50 M NaCl solution. What is the
molarity of NaCl in the final solution?

5. A 10.00 mL of a 0.20 M NaOH solution is added to 10.00 mL of a 0.10 M HCl solution containing
one drop of phenolphthalein indicator. What color change is observed?

6. A 10.00 mL sample of an HCl solution requires 40.00 mL of a 0.25 M NaOH solution to achieve
neutralization. What is the molar concentration of the HCl solution?

NaOH(aq) + HCl(aq)  NaCl(aq) + H2O(l)


P a g e | 185

Source: https:/www.sciencenotes.org/printable-periodic-table/
P a g e | 186

APPENDIX 1.
Prefixes Used with SI
Tera 1012
Giga 109
Mega 106
Kilo 103
Deci 10-1
Centi 10-2
Milli 10-3
Micro 10-6
Nano 10-9
Pico 10-12

APPENDIX 2.
Conversion Factors
1 lb (pound) 453.6 grams
1 gal 3.7854 liters
1 foot 12 inches
1 inch 2.54 cm
1 atm 760 torr or 14.7 psi
1 mile 1609 m
1 Liter-atm 101.3 Joules

Вам также может понравиться